Sunteți pe pagina 1din 46

22

201. The posterior belly of the digastric is supplied


by which of the following nerves?
A. Vagus
B. Facial
C. Trigeminal
D. Cervical spinal
E. Inferior alveolar
202. Which of the following pairs of amino acids is
expected to be found on the interior of a
globular protein?
A. Lysine and arginine
B. Arginine and leucine
C. Leucine and valine
D. Valine and glutamic acid
E. Glutamic acid and lysine
203. Which of the following conditions might be an
initial manifestation of early, acute HIV
infection?
A. Kaposi's sarcoma
B. Wasting syndrome
C. Oral hairy leukoplakia
D. Mononucleosis-like syndrome
E. Pneumocystis carinii pneumonia
204. Which of the following primary teeth has a
distinctly prominent facial cervical ridge that
makes it different from other teeth?
205.
A. Maxillary canine
B. Mandibular canine
C. Mandibular first molar
D. Maxillary second molar
E. Maxillary central incisor
An anemic individual is expected to have
which of the following conditions in systemic
arterial blood?
Oxygen tension
Oxygen content
A.
Elevated Normal
B.
Elevated Reduced
C. Reduced Normal
D.
Normal Reduced
E. Reduced Reduced
206. The horizontal plane diagram below shows
each of the following components of
mandibular movement EXCEPT one. Which
one is this EXCEPTION?
207.
A.
B.
c.
D.
E.
Vertical component of the movement frorr
retruded contact position (1) to
intercuspal position (2)
Horizontal component of movement from
retruded contact position (1) to
intercuspal position (2)
Anterior component of movement from
intercuspal position (2) to maximum
protrusive position (4)
Lateral component of movement from
retruded contact position (1) to
intercuspal position (2)
Mandibular movements occurring
laterotrusively
In the presence of sucrose, Streptococcus
mutans produces
A.
B.
C.
D.
E.
L-forms.
very long chains of cocci.
regular diploid pairs rather than chains.
deposits of a gummy polysaccharide
called glucan.
a very thick wall of muramic acid and
mucopeptide substances.
208. Which of the following properties do mast cells
and basophils have in common?
A. Found in blood
B. Develop from lymphocytes
C. Secrete heparin and histamine
D. Decrease during time of infection
E. Migrate to line the inside of blood
vessels
209. Protein and RNA synthesis occur in each of the
following phases of the cell cycle EXCEPT one.
Which one is this EXCEPTION?
A. Go
B. G
,
C. S
D. G2
E. M
2005 American Dental Association, Joint Commission on National Dental Examinations. All rights reserved. Reprinted by permission.
210. Each of the following is attributable to hepatic
failure EXCEPT one. Which one is this
EXCEPTION?
211.
212.
A.
B.
C.
D.
E.
Tremor
Gynecomastia
Mallory bodies
Hypoalbuminemia
Spider telangiectasia
As viewed from the lingual, the lingual cusp of
a maxillary first premolar is inclined
distally.
mesially.
d istofacially.
mesiolingually.
....
A.
B.
C.
D.
E. directly over the center mesiodistally.
Gram-positive cell walls differ from gram-
negative cell walls in that gram-positive cell
walls uniquely contain
A. teichoic acids.
B. lipopolysaccharides.
C. acid-fast lipids.
D. endotoxins.
213. The main route of calcium excretion from a
normal human adult is
A. urine.
B. sweat.
C. saliva.
D. feces.
E. tears.
214. The parotid duct crosses the
A. temporalis.
B. facial artery.
C. masseter muscle.
D. zygomaticus major.
E. lateral pterygoid.
215. The cusp of Carabelli occurs with sufficient
frequency to be considered normal in which of
the following molars?
A. Primary maxillary first
B. Primary maxillary second
C. Mandibular first
D. Maxillary second
216. Which of the following temporomandibular
joint ligaments restricts the movement of the
disc away from the condyle during.function?
A.
B.
C.
D.
Discal
Stylomandibular
Temporomandibular
Sphenomandibular
217. A patient who has anemia, poorly localized
abdominal pain, and wrist and foot drop
probably is manifesting a toxic state induced
by which of the following?
A. Heroin
B. Cocaine
C. Bismuth
D. Mercury
E. Lead
218. Which of the following is a ketone body?
219.
220.
A. Glycerol
B. Glucagon
C. Acetyl CoA
0, Acetoacetate
E. Phosphatidylcholine
Inorganic crystals in enamel have their long
axes parallel to the rods in which of the
following?
A. Centers of the bodies of the rods only
B. Periphery of the bodi.es of tbe rops
only
C. Bodies of the rods and deviating
increasingly in the tails
D. Tails of the rods and deviating
increasingly from the centers of the
bodies to the periphery
E. Tails of the rods and deviating
increasingly from the periphery of the
bodies to the centers
The occlusal view of a normal dental arch
segment is shown below. The arrow points to
which of the following mandibular teeth?
A. Primary right first molar
B. Primary left first molar
C. Permanent right first premolar
D. Permanent left second premolar
2005 American Dental Association, Joint Commission on National Dental Examinations. All rights reserved. Reprinted by permission.
23
22
201. The posterior belly of the digastric is supplied
by which of the following nerves?
A, Vagus
g, Facial
C, Trigeminal
D, Cervical spinal
E, Inferior alveolar
202. Which of the following pairs of amino acids is
expected to be found on the interior of a
globular protein?
A, Lysine and arginine
B. Arginine and leucine
C, Leucine and valine
D, Valine and glutamic acid
E. Glutamic acid and lysine
203. Which of the following conditions might be an
initial manifestation of early, acute HIV
infection?
A, Kaposi's sarcoma
B. Wasting syndrome
C, Oral hairy leukoplakia
D, Mononucleosis-like syndrome
E, Pneumocystis carinii pneumonia
204. Which of the following primary teeth has a
distinctly prominent facial cervical ridge that
makes it different from other teeth?
205.
A. Maxillary canine
g, Mandibular canine
C, Mandibular first molar
D, Maxillary second molar
E. Maxillary central incisor
An anemic individual is expected to have
which of the following conditions in systemic
arterial blood?
Oxygen tension
Oxygen content
A.
Elevated Normal
g, Elevated Reduced
C, Reduced Normal
D, Normal Reduced
E.
Reduced Reduced
206. The horizontal plane diagram below shows
each of the following components of
mandibular movement EXCEPT one, Which
one is this EXCEPTION?
207.
A.
B,
c,
D,
E.
Vertical component of the movement fror
retruded contact position (1) to
intercuspal position (2)
Horizontal component of movement from
retruded contact position (1) to
intercuspal position (2)
Anterior component of movement from
intercuspal position (2) to maximum
protrusive position (4)
Lateral component of movement from
retruded contact position (1) to
intercuspal position (2)
Mandibular movements occurring
laterotrusively
In the presence of sucrose, Streptococcus
mutans produces
A,
B.
C,
D.
E.
L-forms.
very long chains of cocci.
regular diploid pairs rather than chains.
deposits of a gummy polysaccharide
called glucan.
a very thick wall of muramic acid and
mucopeptide substances.
208. Which of the following properties do mast cells
and basophils have in common?
A. Found in blood
B. Develop from lymphocytes
C. Secrete heparin and histamine
D, Decrease during time of infection
E. Migrate to line the inside of blood
vessels
209. Protein and RNA synthesis occur in each of the
following phases of the cell cycle EXCEPT one,
Which one is this EXCEPTION?
A, Go
B, G,
C, S
D. G2
E. M
2005 American Dental Association, Joint Commission on National Dental Examinations. All rights reserved. Reprinted by permission,
24
221. The presence of mamelons in a 10-year-old
patient suggests which of the following
conditions?
A.
B.
C.
D.
E.
Fluorosis
Malnutrition
Malformation
Anterior open bite
Retained primary teeth
222. Which of the following pathogens is MOST
often associated with infective endocarditis?
A. Pneumococci
B. Group A streptococci
C. Group B streptococci
D. a-hemolytic streptococci
E. streptococci
223. Which of the following coenzymes is required
for the synthesis of pyrimidines, such as
thymine?
A.
B.
C.
D.
E.
Niacin
Thiamine
Folic acid
Riboflavin
Pyridoxine
224. Each of the following venous channels has
direct connections with the pterygoid venous
plexus EXCEPT one. Which one is this
EXCEPTION?
A.
B.
C.
D.
E.
Maxillary vein
Vertebral vein
Deep facial vein
Infraorbital vein
Posterior superior alveolar vein
225. Vitamin C deficiency primarily affects which of
the following tissues?
A.
B.
C.
D.
E.
Hematopoietic
Connective
Epithelial
Muscular
Nervous
226. In the upper limb, which of the following
represents a hallmark of lymphatic vessels?
A.
B.
C.
D.
E.
Contain valves
Follow the veins
Always travel in pairs
Only found on the anterior surface of
the limb
Contain fenestrations to allow passage
of fluids into the interstitium
227. When viewed from the incisal aspect, each of
the following features of an incisor is visible
EXCEPT one. Which one is this EXCEPT/ON?
228.
A. Cingulum
B. Lingual fossa
C. Cervical line
D. Marginal ridges
E. Mesiofacial developmental depression
Each of the following is expected to be active
during fatty acid biosynthesis EXCEPT one.
Which one is this EXCEPT/ON?
A. Tricarboxylic acid cycle
B. Pyruvate dehydrogenase
C. Amino acid catabolism
D. Beta oxidation
E. Glycolysis
229. Pain in inflamed tissues is associated with the
release of which of the following mediators?
A.
B.
C.
D.
E.
Histamine
Serotonin
Bradykinin
Leukotrienes
Complement components
230. Which of the following cranial nerves contains
parasympathetic components?
A.
B.
C.
D.
E.
Facial
Abducens
Trigeminal
Hypoglossal
Spinal accessory
2005 American Dental Association J . t c .. N' I D '" . . .
,Oln ommlsslon on atlona ental Examinations. All nghts reserved. Repnnted by permission.

231. In an ideal intercuspal position, the mesiofacial
groove of a mandibular first molar opposes
which part of the maxillary first molar?
232.
233.
234.
A.
B.
C.
D.
Tip of the distofacial cusp
Tip of the mesiofacial cusp
Triangular ridge of the distofacial cusp
Triangular ridge of the mesiofacial cusp
Which of the following represents genes that
often encode proteins necessary for antibiotic
resistance, and that can change positions on a
chromosome or "jump" from a plasmid to a
chromosome?
A. Tryptophan operon
B. Lysogens
C. Prophage
D. Transposons
Which of the following BEST describes the
major function of histones?
A. Unwind DNA.
B. Activate genes.
C. Stabilize DNA in a compact form.
D. Remove exons during RNA splicing.
E. Stabilize RNA during transcription.
The thyrocervical trunk is comprised of the
inferior thyroid, the transverse cervical artery,
and the
A.
B.
C.
D.
E.
vertebral.
suprascapular.
dorsal scapular.
superior thyroid.
ascending pharyngeal.
235. Under physiologic conditions, the LOWEST
partial pressure of oxygen is found in
236.
A. expired air.
B. alveolar air.
C. venous blood.
D. arterial blood.
E. normal atmosphere air.
The condyle on the laterotrusive side
generally rotates about a
A. sagittal axis only.
B. horizontal axis only.
C. horizontal axis and translates laterally.
D. vertical axis and translates later01ly.
237. Each of the following diseases is transmitted
from person to person by droplets or droplet
spray EXCEPT one. Which one is this
EXCEPT/ON?
238.
239.
A.
B.
C.
D.
E.
Condyloma acuminatum
Whooping cough
Meningitis
Diphtheria
Pneumonia
Which of the following bones forms the roof of
the orbit?
A. Zygomatic
B. Maxilla
C. Palatine
D. Sphenoid
E. Frontal
Which of the following describes sebaceous
glands of the skin?
A. They are associated with hair follicles
and are derived from ectoderm.
B. They serve a secretory function in
conjunction with Pacinian corpuscles.
C. They secrete a serous material by the
merocrine method.
D. They yield a salty secretion and are
derived from mesoderm.
240. The first step in the pathway for fatty acid
synthesis is catalyzed by an allosteric enzyme
which is the principal regulator of the pathway.
This enzyme is
A. thiolase.
B. pyruvate carboxylase.
C. citrate synthetase.
D. acetyl CoA carboxylase.
E. pyruvate dehydrogenase.
241. Which of the following is a major disadvantage
of ethylene oxide sterilization?
A.
B.
C.
D.
E.
Difficult to obtain
Very toxic to humans
Requires high temperatures (160)
Causes corrosion of metal instruments
Causes discoloration of clear plastic
objects
2005 American Dental Association, Joint Commission on National Dental Examinations. All rights reserved. Reprinted by permission.
25
26
242. What teeth occlude with only one opposing
tooth?
A. Maxillary canines
B. Mandibular lateral incisors
C. Mandibular central incisors
D. Maxillary first premolars
E. Mandibular third molars
243. Endochondral ossification occurs in the
formation of
A.
B.
C.
D.
E.
all bones.
flat bones.
long bones.
elastic cartilage.
mesenchymal sheets.
~
244. Which of the following represents the MOST
common cause of bronchiolitis and pneumonia
in infants?
A. Measles virus
B. Cytomegalovirus
C. Influenza virus
D. Herpes simplex virus
E. Respiratory syncytial virus
245. The developmental groove between the
distofacial cusp and the distal cusp of a
mandibular first molar is the
A.
B.
C.
D.
E.
facial.
distal.
distofacial.
mesiofacial.
distal oblique.
246. Which of the following represents the anterior
boundary of the mandibular fossa of the
temporal bone?
A. Styloid process
B. Articular eminence
C. Petrotympanic fissure
D. External auditory meatus
E. Mastoid process of temporal bone
247. The development of which of the following
types of hepatitis depends on the host having
been previously infected with hepatitis B
virus?
A. Hepatitis A
B. Hepatitis C
C. Hepatitis D
D. Hepatitis E
E. Hepatitis G
248. Glucose-6-phosphatase is found associated
wifh which of the following?
A.
B.
C.
D.
Kidneys and liver
Liver and muscles
Kidneys and brain
Kidneys and muscles
249. The osteoclast originates from which of the
following?
A. Monocyte
B. Osteocyte
C. Osteoblast
D. Neutrophil
E. Bone-lining cell
250. Which of the following permanent roots is thin
mesiodistally, wide faciolingually, and concave
on both mesial and distal surfaces?
A.
B.
C.
D.
E.
Maxillary central incisor
Mandibular central incisor
Mandibular second premolar
Facial root of a maxillary first premolar
Lingual root of a maxillary first molar
251. The catabolism of which of the following results
in no energy production in the form of ATP?
A. Lipid
B. Protein
C. Nucleotide
D. Carbohydrate
252. Recovery from an infection with mumps virus
confers lifelong immunity. This describes
which of the following types of immunity?
A.
B.
C.
D.
E.
Innate
Natural active
Natural passive
Arlificial active
Arlificial passive
2005 American Dental Association, Joint Commission on National Dental Examinations. All rights reserved. Reprinted by permission.
253. Which of the following represents the structure
in the maxillary alveolar bone that maxillary
premolar roots occasionally penetrate?
254.
A.
B.
C.
D.
E,
Antrum
Nasal septum
Frontal sinus
Zygomatic arch
Mandibular fossa
The mylohyoid ridge (line) is found on which
of the following portions of the mandible?
A. Coronoid
B. Neck
C. Ramus
D. Condyle
E. Body
255. Which of the following represents the portion
of the bacterial endotoxin molecule that is
MOST responsible for its toxic activity?
A.
B.
C.
D.
Lipid A
Protein A
o antigen
Core polysaccharide
256. Which of the following BEST describes
restriction enzymes?
A.
B.
C.
D.
E.
Exonucleases
Topoisomerases
Enzymes that degrade RNA
Non-specific endonucleases
Site-specific endonucleases
257. The contact areas of anterior teeth are incisal
to the middle thirds in each of the following
EXCEPT one. Which one is this EXCEPTION?
A.
B.
C.
D.
E.
Distal contact of the maxillary canine
Mesial contact of the maxillary canine
Mesial contact of the maxillary lateral
incisor
Mesial contact of the mandibular lateral
incisor
Distal contact of the mandibular-lateral
incisor
258. Proteins for extracellular use can be
synthesized in which of the following
structu res?
A.
B.
C.
D.
E.
Nucleolemma
Mitochondria
Heterochromatin
Rough endoplasmic reticulum
Smooth endoplasmic reticulum
259. Which of the following types of blotting can be
used to identify DNA restriction fragments?
260.
A. Eastern
B. Southern
C. Northern
D. Western
The unique amino acid composition of
collagen is reflected in the high content of
A.
B.
C.
D.
E.
valine, threonine, and lysine.
desmosine, glycine, and proline.
cysteine, tyrosine, and phenylalanine.
glycine, proline, and hydroxyproline.
cysteine, hydroxylysine, and proline.
261. Which of the following represents the type of
maxillofacial bone graft having the BEST
chance of success?
A. Isograft
B. Homograft
r-
Allograft v.
D. Autograft
E. Xenograft
262. Which of the following represents a risk factor
for the development of endometrial carcinoma?
A.
B.
C.
D.
E.
Herpes simplex virus infection
Endometriosis
Multiple leiomyomas
Multiple pregnancies
Hyperestrogenism
263. Which of the following vitamins is essential for
the normal elaboration and maintenance of
bone matrix, cartilage, and dentin?
A.
B.
C.
D.
Niacin
Vitamin E
Ascorbic acid
Pantothenic acid
2005 American Dental Association, Joint Commission on National Dental Examinations. All rights reserved. Reprinted by permission.
27
28
264. Calcium stimulates muscle contraction. Its
effect is mediated through binding to which of
the following?
A.
B.
C.
D.
E.
Actin
Tropomyosin
Troponin I
Troponin C
Sarcoplasmic reticulum
265. Which of the following features helps to
distinguish a maxillary lateral incisor from a
mandibular lateral incisor? The maxillary
lateral has
266.
267.
A. a thinner incisal ridge.
B. a crown more twisted on the root.
C. a more pronounced lingual fossa.
D. a root that is flattened mesiodistally.
E. a mesioincisal angle that is sharper.
The arrow in the illustration below represents
the path taken by an opposing cusp. Which of
the following is the cusp and mandibular
movement indicated by the arrow?
A.
B.
c.
D.
Facial cusp of a second premolar,
retrusive
Mesiofacial cusp of a first molar,
protrusive
Mesiofacial cusp of a second molar,
laterotrusive
Distofacial cusp of a first molar,
protrusive
Which of the following produce surfactant?
A. Monocytes
B. Clara cells
C. Goblet cells
D. Type I pneumocytes
E. Type" pneumocytes
268. Each of the following fluids is considered one
that can transmit HIV EXCEPT one. Which
one is this EXCEPTION?
A.
B.
C.
D.
E.
Semen
Serum
Saliva
Amniotic fluid
Breast milk
259. When compared to a maxillary first molar, the
roots of a maxillary second molar
A.
B.
C.
D.
E.
are longer.
are more divergent.
are fewer in number.
have less potential for fusion.
are greater in distal inclination.
270. In right laterotrusive movement, the lingual
cusp of a maxillary right second premolar
passes through which of the following
mandibular structures?
A.
B.
C.
D.
Facial groove of the right first molar
Lingual groove of the right first molar
Embrasure between the right first
premolar and the right second
premolar
Embrasure between the right second
premolar and the right first molar
271. Which of the following is the MOST appropriate
sterilization method for a heatlabile item to be
used in a surgical procedure?
272.
A. Dry heat
B. Autoclaving
C. Ethylene oxide
D. Saturated chemical vapor
E. Immersion in chemical sterilant
The primary function of the anterior portion of
the temporalis muscle is to
A.
B.
C.
D.
E.
elevate the mandible only.
protrude the mandible only.
elevate and retrude the mandible.
help stabilize the articular disc.
move the mandible to the contralateral
side.
273. Bile salts are sodium salts of bile acid that are
conjugated with
A.
B.
C.
D.
cysteine and serine.
taurine and glycine.
methionine and lysine.
homocysteine and ornithine.
2005 American Dental Association, Joint Commission on National Dental Examinations. All rights reserved. Fleprinted by permission.
274.
275.
276.
277.
278.
In order for RNA oncogenic viruses to be
integrated into the host genome, they must
possess
A. translation enzymes.
B. restriction endonuclease.
C. DNA-dependent RNA polymerase.
D. RNA-dependent DNA polymerase.
On the lingual crown surface of maxillary
canines, which of the following is the structure
located immediately mesial to the distolingual
fossa?
A. Cingulum
B. Lingual ridge
C. Mesiolingual fossa
D. Mesial marginal ridge
E. Distal marginal ridge
In which of the following structures are
glycoproteins assembled for extracellular use?
A. Nucleolemma
B. Polyribosomes
C. Golgi apparatus
D. Rough endoplasmic reticulum
E. Smooth endoplasmic reticulum
Which of the following statements BEST
describes the strength of cardiac muscle
contraction? It is
A. decreased when extracellular Ca++ is
increased.
B. increased when extracellular Ca++ is
increased.
C. mainly regulated by vagal impulses.
D. unaffected by extracellular Ca++'
E. mainly regulated by heart rate.
Oxygen-dependent killing of bacteria by
neutrophils involves each of the following
EXCEPT one. Which one is this
EXCEPTION?
A. Superoxide
B. Collagenase
C. Myeloperoxidase
D. Hydrogen peroxide
E. NADP dehydrogenase
279.
280.
281.
282.
283.
29
Which of the following are the teeth whose
masticatory function is primarily biting?
A. Incisors and canines
B. Incisors and premolars
C. Canines and premolars
D. Premolars and molars
Which of the following represents the
predominant serum immunoglobulin found
during a memory response?
A. IgA
B. IgD
C. IgE
D. IgG
E. s-lgA
Which of the following mechanisms operates
in liver cells to regulate breakdown of
glycogen?
A. Zymogen activation
B. Phosphorylation of phosphorylase b to
phosphorylase a
C. Induction of phosphorylase b by an
inducer
. D. Inhibition of adenylate cyclase
Which of the following structures can be
palpated by way of the external auditory
(acoustic) meatus?
A. Zygomatic arch
B. Mandibular notch
C. Lateral pterygoid muscle
D. Coronoid process of the mandible
E. Posterior aspect of the mandibular
condyle
Which main product of protein nitrogen
metabolism is found in human urine?
A. Urea
B. Ammonia
C. Creatine
D. Uric acid
E. Creatinine
2005 American Dental Association, Joint Commission on National Dental Examinations. All rights reserved. Reprinted by permission.
30
284. Which of the follmving nerves travels from the
carotid triangle into the submandibular triangle
of the neck?
A. Lingual
B. Hypoglossal
C. Great auricular
D. Ansa cervicalis
E. Recurrent laryngeal
285. Which of the following best represents the
structure on the crown of the maxillary canine
which normally lies directly distal to the
mesiolingual fossa?
A.
B.
C.
D.
E.
Lingual ridge
Distal fossa
Distolingual fossa
Distal marginal ridge
Mesial marginal ridge
286. Which of the following organisms causes a
granulomatous, tuberculosis-like infection?
A.
B.
C.
D.
E.
Aspergillus fumigatus
Microsporum audouini
Histoplasma capsula tum
Epidermophyton floccosum
Trichophyton schoenleinii
287. Stimulation of the posterior hypothalamus by a
reduction in core temperature will produce
which of the following?
288.
A. Panting
B. Sweating
C. Vomiting
D. Shivering
E. Vasodilation
Each of the following structures is found in the
posterior mediastinum EXCEPT one. Which
one is this EXCEPTION?
A.
B.
C.
D.
E.
Vagus nerve
Phrenic nerve
Thoracic duct
Greater splanchnic nerve
Sympathetic chain ganglia
289. When an adult with normal occlusion moves
the mandible from maximum intercuspation to
right lateral relation, which of the following
cusps of maxillary left teeth moves through the
facial embrasure between the mandibular left
premolars?
Cusp
Tooth
A.
Lingual
First premolar
B.
Facial
Second premolar
C.
Lingual
Second premolar
D.
Mesiofacial
First molar
E.
Distofacial
First molar
290. A patient has a periapical abscess and a
history of hypersensitivity to penicillin. Which
of the following antimicrobial agents might be
the preferred alternative for this patient?
A.
B.
C.
D.
c
L.
Amoxicillin
Azith romycin
Tetracycline
Cephalosporin
Nystatin
291. Which of the following compounds is a
precursor of the pigments of the skin, hair,
and eyes?
A.
B.
C.
D.
E.
Tyrosine
Cysteine
Lysine
Creatine
Galactosamine
292. Intelligence and sensory-motor functions of a
patient appear to be intact. However, the
patient lacks self-discipline and has exhibited
anti-social behavior. He is unable to plan for
the future or to organize behaviors into logical
sequences. A lesion in which of the following
areas is likely?
A.
B.
C.
D.
E.
Frontal lobe
Occipital lobe
Internal capsule
Temporal lobe
Parietal lobe
2005 American Dental Association, Joint Commission on National Dental Examinations. All rights reserved. Reprinted by permission.
293. Which of the following organs secretes insulin-
like growth factor 1 (IGF-1), and what
stimulates this secretion?
294.
295.
296.
A.
B.
Anterior pituitary, increasing IGF-RH in
the blood
Anterior pituitary, increasing GRH in the
blood
C. Kidney, increasing growth hormone in
the blood
D. Liver, increasing growth hormone in the
blood
E. Liver, increasing TSH in the blood
Edema as a result of a hemodynamic disorder
commonly occurs in each of the following
locations EXCEPT one. Which one is this
EXCEPTION?
A. Lungs
B. Brain
C. Subcutaneous tissues
D. Peritoneal cavity
E. Pancreas
The tissue present between the articulating
bones of the temporomandibular joint is called
the
A.
B.
C.
D.
E.
articular disc.
fibrous capsule.
synovial membrane.
joint capsule.
bilaminar zone.
The progression from a healthy gingival
sulcus to gingivitis is associated with a shift
towards
A. gram-positive cocci.
B. gram-negative anaerobic rods.
C. gram-positive filamentous rods.
D. gram-negative anaerobic cocci.
E. gram-positive facultative cocci.
297. Which of the following enzymes synthesizes
polynucleotide chains from nucleotides and
does NOT require a primer chain?
298.
299.
A.
B.
C.
D.
E.
Ligase
Exonuclease
Endonuclease
DNA polymerase
RNA polymerase
Which of the following structures travels just
posterior to the arch of the azygos vein?
A. Hemiazygos vein
B. Right vagus nerve
C. Left vagus nerve
D. Right lymphatic duct
E. Greater splanchnic nerve
An infection spreading by way of the lymphatic
system from the vermiform appendix first enters
the blood stream at the
A.
B.
C.
D.
E.
renal vein.
appendicular vein.
inferior vena cava.
brachiocephalic vein.
internal thoracic vein.
300. Which of the following describes a major
effect of sickle cell anemia?
A.
B.
C.
D.
E.
Absence of biphosphoglycerate binding of
hemoglobin
Substitution of 2 proximal histidines
Decreased solubility of the deoxy form of
hemoglobin
A P
50
value for hemoglobin similar to that
of myoglobin
Decreased number of subunits in
hemoglobin
2005 American Dental Association, Joint Commission on National Dental Examinations. All rights reserved. Reprinted by permission.
31
301. Human infection with Histoplasma capsula tum
is usually a result of
302.
303.
304.
A.
B.
C.
D.
E.
inhaling contaminated air.
drinking contaminated water.
ingesting contaminated food.
direct human-to-human contact.
direct animal-to-human contact.
From a frontal view, which of the following
describes the plane of occlusion of the
mandibular arch in a normal dentition?
A. It is flat.
B. It is a convex curve.
C. It is a concave curve.
D. It is the result of uniform perpendicular
alignment of the teeth.
An abnormal increase in blood pressure in a
healthy person will RESUL Tin an increased
number of impulses traveling to the heart ove!"
which of the following nerves?
A. Vagus nerves
B. Sympathetic nerves
C. Branches of thoracic nerves
D. Branches of the trigeminal nerve
E. Branches of the glossopharyngeal
nerve
Which of the following hormones exerts the
LEAST effect on calcium metabolism of bone
tissue?
A.
B.
C.
D.
E.
Androgen
Estrogen
Norepinephrine
Thyroid hormone
Parathyroid hormone
305. Which of the following types of bronchogenic
carcinoma is MOST commonly associated
with paraneoplastic syndrome?
A.
B.
C.
D.
Small cell carcinoma
Large cell carcinoma
Mesothelioma
Adenocarcinoma
306. Which of the following teeth normally has the
longest root?
A.
B.
C.
D.
E.
Maxillary canine
Mandibular canine
Maxillary central incisor
Mandibular first premolar
Maxillary second premolar
307. Tooth erosion in bulimic patients is due to
308.
309.
A. hyposalivation.
B. hypersalivation.
C. action of pepsin.
D. excessive fluoride intake.
E. solubility of hydroxyapatite in acid.
Which of the following represents the basis for
the topical application of fluorides in dental
caries prevention?
A.
B.
C.
D.
E.
Fluoride penetrates the enamel through
the lamellae.
Keratin content of the enamel is made
more insoluble.
Fluoride coats the tooth forming a
uniform protective barrier.
The primary cuticle. being less
calcified, absorbs the fluoride.
Acid solubility of the surface enamel is
reduced by the fluoride.
Which of the following statements describes
an exampl0 of innate immunity?
A.
B.
C.
D.
E.
An allergic reaction to insect venom
The classical pathway of complement
The destruction of virus-infected cells
by T-killer cells
The production of IgG in response to
insect venom
Phagocytosis of microbes by
neutrophils and macrophages
310. Which of the following mandibular molar
proximal crown surfaces normally exhibits the
LARGEST size?
A.
B.
C.
D.
E.
Mesial of the first
Distal of the first
Mesial of the second
Distal of the second
Mesial of the third
2005 American Dental Association, Joint Commission on National Dental Examinations. All rights reserved. Reprinted by permission.
311.
312.
In a CUSP TO FOSSA contacting relationship in
intercuspal position, the maxillary first premolar
is MOST likely to articulate with which of the
following mandibular teeth?
A. Lateral incisor
B. Lateral incisor and canine
C. First premolar
D. Canine and first premolar
E. First premolar and second premolar
Which of the following has the LEAST ability
to regenerate?
A.
B.
C.
D.
E.
Bone
Liver
Striated muscle
Collagen
Smooth muscle
313. Which of the following represents normal
glucose clearance?
314.
315.
316.
A. 0 mg/min
B. 44 mg/min
C. 125 mg/min
D. 600 mg/min
There is a distinct change in the type of
surface epithelium at the junction of the
A. ileum and colon.
B. fundus and pylorus.
C. duodenum and jejunum.
D. stomach and esophagus.
E. oropharynx and esophagus.
A patient bites down rapidly on an unexpected
hard surface while chewing. Cessation of motor
unit recruitment in jaw closing muscles is
caused by stimulation of
A. muscle spindles.
B. mucosal mechanoreceptors.
C. periodontal mechanoreceptors.
D. nociceptors in the dental pulp.
Pulmonary edema and filling of the pleural
cavity by a transudate is MOST suggestive of
which of the following underlying diseases?
A. Cor pulmonale
B. Pulmonary embolism
C. Pulmonic valve stenosis
D. Tricuspid valve prolapse
E. Left-sided heart failure
317.
318.
The major grooves of the mandibular second
molar form which of the following patterns?
A. H
B. Y
C. X
D. +
E.
If the maxillary and mandibular incisors are in
crossbite, then the contacting surfaces, during
maximum intercuspation, would be
A.
B.
C.
D.
mandibular lingual and maxillary facial.
mandibular facial and maxillary facial.
mandibular lingual and maxillary
lingual.
mandibular facial and maxillary lingual.
319. Which of the following disorders is X-linked
recessive?
320.
321.
A.
B.
C.
D.
E.
Neurofibromatosis
Turner syndrome
Cystic fibrosis
Marfan syndrome
Hemophilia A
Active transport systems generally involve
specific binding molecules that are classified
as a
A. lipid.
B. protein.
C. lipoprotein.
D. carbohydrate.
E. phospholipid.
Which of the following structures directly
develops from the cells remaining in the
remnants of the preovulatory follicle after
ovulation?
A. Theca interna
B. Corpus luteum
C. Corpus albicans
D. Atretic follicle
E. Interstitial gland
2005 American Dental Association, Joint Commission on National Dental Examinations. All rights reserved. Reprinted by permission.
33
34
322. Which of the following provides the immediate
energy source for the transport of glucose into
intestinal epithelial cells?
A.
B.
C.
D.
NADH
NADPH
Na+ gradient across the luminal
membrane
H+ gradient across the luminal
membrane
323. The biologic effect of exotoxin A produced by
Pseudomollus aeruginosa is to
324.
325.
326.
A. increase membrane permeability,
B. inactivate IgG.
C. inhibit protein synthesis.
D. increase levels of cAMP.
E. inhibit DNA synthesis.
As distinguished from a maxillary canine, a
mandibular canine typically has
A. a sharper facial cusp tip.
B. contact areas located more incisally.
C. a more pronounced cingulum.
D. a more convex mesial border, when
viewed facially.
Urolithiasis is associated with each of the
following conditions EXCEPT one. Which one
is this EXCEPTION?
A. Gout
B. Hypertension
C. Hypercalcemia
D. Renal infection
E. Hyperparathyroidism
An action potential initiated at the midpoint
along the length of an axon will
A. not propagate.
B. propagate towards the soma.
C. propagate towards the nerve ending.
D. propagate towards both the soma and
the nerve ending.
327. In which of the following are tonofibrils and
desmosomes especially well-developed?
A.
Neurons
B.
Fibroblasts
C.
Ameloblasts
D.
Odontoblasts
E.
Keratinocytes
328. Which of the following cells are responsible for
delayed-type hypersensitivity (Type IV
hypersensitivity) ?
329.
330.
331.
332.
A.
B.
C.
D.
E.
B cells, Th2 cells
B cells, neutrophils
Th1 cells, macrophages
Mast cells, eosinophils
IgE, mast cells, histamine
Which of the following teeth has its mesial
marginal ridge located more cervically than its
distal marginal ridge?
A. Mandibular first premolar
B. Maxillary first premolar
C. Maxillary first molar
D. Primary mandibular first molar
E. Mandibular second molar
Which of the following represents the ridge on
the occlusal surface of the maxillary first molar
that normally forms the distal boundary of the
central fossa?
A. Oblique
B. Distal marginal
C. Mesial marginal
D. Mesial cusp ridges of the MF cusp
E. Distal cusp ridges of the OF cusp
Which of the following characterizes a benign
tumor arising from skeletal muscle?
A. Leiomyoma
B. Papilloma
C. Rhabdomyoma
D. Leiomyosarcoma
E. Rhabdomyosarcoma
The dentin of the crown of a tooth and that of
the root differ in which of the following ways?
A.
B.
C.
D.
E.
Root dentin contains lacunae.
Root dentin is more highly calcified.
Root dentin contains more tubules per
given area.
Granular layer of Tomes is found
primarily in the crown.
Primary curvatures of the tubules
decrease in root dentin.
2005 American Dental Association, Joint Commission on National Dental Examinations. All rights reserved. Reprinted by permission.
333. Which of the following explains why an
increased filling of the ventricle during diastole
causes a more forceful heart beat? The
increased ventricular volume
A.
B.
C.
D.
diminishes the refractory period of the
ventricle.
increases end-diastolic fiber length.
facilitates conduction in the heart.
produces a less than optimal load.
334. Following prophylactic administration of
amoxicillin, a patient becomes hypotensive
and itchy and is having difficulty breathing.
Which of the following is MOST probably
occurring?
335.
336.
A. CD4 lymphocytes are secreting
Iymphokines resulting in edema.
B. Amoxicillin reacts with IgE and
activates cytotoxic T cells to release
Iymphokines.
C. IgG and complement are inducing the
chemotaxis of neutrophils out of
vessels.
D. IgE bound to antigen results in
histamine release from mast cells,
A transverse ridge results from the union of
A. Mesial and distal marginal ridges
B. Facial and lingual cusp ridges
C. Facial and lingual triangular ridges
D. Marginal and triangular
E. Cusp ridge and triangular ridge
Which of the following is MOST likely to
precede an impending atherosclerotic cerebral
infarction?
A. Angina pectoris
B. Pulmonary embolus
C. Myocardial infarction
D. Ruptured berry aneurysm
E. Transient ischemic attacks
337. The biologically active conformation of trimeric
G-proteins requires
A. the alpha-subunit to bind GDP.
B. the alpha-subunit to bind GTP.
C. the hydrolysis of BetaGamma-subunits.
D. the alpha-subunit to phosphorylate
downstream targets.
338. During the life span of a multi rooted tooth,
dentin continues to form MOST rapidly at
which of the following locations?
A.
B.
C.
D.
E.
Within the root canals
At the dentinoenamel junction
Floor and roof of the pulp chamber
Mesial and distal walls of the pulp
chamber
Facial and lingual walls of the pulp
chamber
339. The glomerular lesions in immune complex
disease result from deposition of which of the
following?
A. IgE
B. IgG
C. TCR
D. Histamine
E. Interleukin
340. Which of the following is normally exhibited by
maxillary second premolars?
A. Two roots
B. A mesial concavity of the crown
C. A facial crown contour that is greatly
inclined to the lingual
D. A lingual height of contour in the
cervical third of the crown
E. A crown that is wider faciolingually
than mesiodistally
341. The articular disc of the temporomandibular
joint consists of
A.
8.
C.
D.
E.
hyaiine cartilage.
loose fibrous connective tissue.
an outer synovial layer and an inner
fibrous layer.
dense fibrous connective tissue.
an outer layer of mesothelium and an
inner core of hyaline cartilage.
342. Which of the following diseases is caused by
an agent that produces neither exotoxins nor
endotoxins?
A.
Cholera
B. Tetanus
C. Brucellosis
D. Gas gangrene
E. Tuberculosis
2005 American Dental Association, Joint Commission on National Dental Examinations. Ali rights reserved. Reprinted by permission.
35
36
343. Which of the following teeth might possess
three cusps?
A.
B.
C.
D.
E.
Maxillary second premolar and maxillary
first molar
Maxillary second premolar and
mandibular first premolar
Maxillary first molar and mandibular
second molar
Maxillary second molar and mandibular
first premolar
Maxillary second molar and mandibular
second premolar
: ~ 4 4 . Arneloblasts will form enamel if which of the
following cells are present?
A.
8.
C.
D.
E.
Stellate reticulum
Stratum intermedium
Langerhans and Merkel
Outer enamel epithelium
Reduced enamel epithelium
345. A mandibular first molar usually has how
many roots and canals?
Roots Canals
A. Two Two
B. Two Three
C. Three Two
D. Three Three
346. In osteomalacia, bones are weak because of
the failure of
A.
B.
C.
D.
E.
remodeling of bone.
bone matrix formation.
osteoblast proliferation.
bone matrix calcification.
interstitial growth of cartilage.
347. The kidney dialysis machine is an example of
which of the following mechanical systems?
A. Sorting
B. Cotransport
C. Countercurrent
D. Active transport
348. The OLDEST enamel in a fully erupted first
molar is located at the
A.
B.
C.
D.
E.
cervix of the tooth.
oral surface of a cusp.
mesial and distal surfaces of the tooth.
dentinoenamel junction underlying a
cusp.
dentinoenamel junction underlying a
fissure.
349. Which of the following roots MOST likely has
two canals?
A.
B.
C.
D.
E.
Facial root of the maxillary first premolar
Palatal root of the maxillary first molar
Distofacial root of the maxillary first
molar
Mesiofacial root of the maxillary first
molar
Distal root of the mandibular first molar
350. Which of the following represents the term
"characteristic of a single disease"?
351.
A. Pathogenic
B. Pathologic
C. . Idiopathic
D. Iatrogenic
E. Pathognomonic
Each of the following statements correctly
describes similar characteristics of the
chemical senses of taste and smell EXCEPT
one. Which one is this EXCEPTION?
A. Receptor cells are replaced regularly.
B. There are primary classes of taste and
of odor.
C.
D.
E.
Receptors are located on cilia or
microvilli at the apical ends of cells.
Receptors initiate action potentials
directly to respective cranial nerve
sensory fibers.
Molecules must be dissolved in saliva
or mucus to interact with receptor
membrane proteins.
2005 American Dental Association, Joint Commission on National Dental Examinations. All rights reserved. Reprinted by permission.
38
Test item #'5 362-368 refer to the following testlet.
A 42-year-old female with a history of
hyperparathyroidism presents with a complaint of
bilateral pain in the temporomandibular joint on
closing. She reports that her ears are occasionally
"clogged", and she sometimes experiences a "ringing"
sensation. The dentist determines that these
symptoms are related to her TMJ condition.
362. Given her reported hyperparathyroidism, this
patient is likely to show each of the following
signs or symptoms EXCEPT one. Which one
is this EXCEPTION?
A. Urolithiasis
B. Elevated serum calcium
C. Tetanic muscular convulsions
D. Central giant-cell bone lesions
E. Loss of lamina dura surrounding
multiple teeth
363. Which of the following will most likely be
INCREASED in this patient?
A. Intestinal calcium absorption
B. Intestinal organic phosphate absorption
C. Renal calcitriol production
D. Renal calcium excretion
E. Renal organic phosphate excretion
364. This patient will MOST likely be predisposed
to which of the following?
A.
B.
C.
D.
Muscle spasm
Reduced serum Ca++ levels
Increased osteoblastic activity
Increased likelihood of bone fracture
365. Examination reveals diminished parotid
salivation due to middle ear involvement.
Which of the following neNes is MOST likely
involved?
A. Facial
B. Lesser petrosal
C. Greater petrosal
D. Auriculotemporal
E. Chorda tympanic
366. In this patient, which of the following region
of the articular surface of the TMJ is unlikel
to be present?
367.
368.
A. Calcified cartilage
B. Subarticular bone
C. Proliferative zone
D. Fibrous a r t i ~ u l a r layer
E. Fibrocartilaginous zone
If the patient's sensation of "clogged ears" is
due to pressure on the auditory tubes, then
which of the following nerves is providing this
sensory innervation?
A. Glossopharyngeal
B. Lesser occipital
C. Vestibulocochlear
D. Zygomatic temporal
E. Posterior deep temporal
This patient's feeling that her ears are
"clogged" might be due to restricted function
of which of the following muscles?
A. Digastric
B. Omohyoid
C. Temporalis
D. Buccinator Tensor
E. veli palatini
Test item #'s 369-377 refer to the following testlet
A lO-year-old female with a history of acute
lymphoblastic leukemia is undergoing orthodontic
treatment that is complicated by poor oral hygiene.
Oral examination reveals generalized ceNical plaque
and enlargement of the gingiva. There is generalized
bleeding on probing.
369. Each of the following statements correctly
describes a clinical feature of this patient's
leukemia EXCEPT one. Which one is this
EXCEPTION?
A.
B.
C.
D.
Multiple, acutely tender lymph nodes
due to central node ischemic necrosis.
CNS infiltration related to headache,
vomiting, and palsies.
Bone pain secondary to leukemia
infiltration of the marrow and
periosteum.
Symptoms related to bone marrow
suppression, including anemia and
thrombocytopenia.
2005 American Dental Association, Joint Commission on National Dental Examinations. All rights reserved. Reprinted by permission.
40
Test item #'s 378-389 refer to the following testlet.
An 80-year-old female with a history of cardiovascular
disease presents with a complaint of pain under her
complete dentures when biting. She has worn them for
15 years, but seldom removes or cleans them.
Removal of the dentures reveals diffuse erythema of
underlying mucosal tissue. Bone resorption is noted
from detectable reduction in height of the alveolar
ridges due to the ill-fitting denture.
378. Each of the following statements correctly
describes myocardial infarction (MI) EXCEPT
one. Which one is this EXCEPT/ON?
379.
380.
A.
B.
C.
D.
E.
Most acute Mis are caused by coronary
artery th!"Ombosis.
Acute MI is the most common cause of
death in industrialized nations.
Pain f!"Om MI can usually be relieved by
vasodilators such as nitroglycerin.
Thrombolytic agents such as
streptokinase often limit the size of
infarction.
Myocardial necrosis usually begins 20-
30 minutes after coronary artery
occlusion.
When fully reclined in the dental chair, the
patient reports difficulty in breathing. This is
MOST likely the direct result of which of the
following?
A. Hypoxia
B. Angina pectoris
C. Bronchoconstriction
D. Carbon dioxide retention
E. Congestive heart failure
The atherosclerotic condition of this patient's
arteries likely involves hypertrophy of which of
the following?
A.
B.
C.
D.
Tunica media
Tunica intima
Tunica adventitia
Internal elastic membrane
381. Each of ihe following statements describes a
characteristic feature of ischemic heart
disease EXCEPT one. Which one is this
EXCEPT/ON?
A
B.
C.
D.
Usually results from complete
occlusion of one or more coronary
arteries.
Represents an imbalance between
myocardial oxygen demand and
available blood supply.
Has a peak incidence in men over 60
years of age, and women over 70
years of age.
Contributing factors include chronic
hypertension and high levels of low-
density lipoproteins.
382. Which of the following represents the MOST
likely pathologic change in this patient's heart?
383.
384.
385.
A. Vegetations of the aortic valve
B. Severe mitral valve thickening
C. Hypert!"Ophy of the left ventricle
D. f\lecrosis in the right ventricle
E. Scarring in the left ventricle
The patient's cardiovascular disease has
resulted in chronic atrial fibrillation. Her
physician has prescribed warfarin
(Coumadin) MOST likely to prevent
A. st!"Oke.
B. hypertension.
C. atherosclerosis.
D. sudden cardiac death.
E. left ventricular hypertrophy.
The bone resorption seen in elderly patients
with low dietary calcium is enhanced by which
of the following hormones?
A. Insulin
B. Estrogen
C. Aldosterone
D. Thyroid stimulating hormone
E. Parathyroid hormone
Administration of a local anesthetic with
epinephrine will MOST likely produce which of
the following cardiovascular effects?
A.
B.
C.
D.
Increased heart rate
Decreased heart rate
Increased diastolic blood pressure
Decreased systolic blood pressure
2005 American Dental Association, Joint Commission on National Dental Examinations. All rights reserved. Reprinted by permission.
386. To minimize pain when biting, the patient
moves her mandible to the right and chews
only on that side. Which of the following
muscles primarily produces this mandibular
movement?
A. Right masseter
B. Left temporalis
C. Right medial pterygoid
D. Right lateral pterygoid
E. Left lateral pterygoid
387. Which of the following BEST describes the
t'esult of continued abrasion of the dentures
on the underlying alveolar mucosa?
388.
A. Lining mucosa becoming gingival mucosa
B. Lining mucosa becoming masticatory
mucosa
C. Gingival mucosa becoming
orthokeratinized
D.
E.
Masticatory mucosa becoming
parakeratinized
Orthokeratinized mucosa becoming
parake rati n ized
The patient's mandibular denture is determined
to be pressing too far inferiorly, so that
contraction of the muscular floor renders the
denture unusable. Which of the following
muscles is responsible for this circumstance?
A.
B.
C.
D.
E.
Mentalis
Mylohyoid
Geniohyoid
Genioglossus
Medial pterygoid
389. The new denture teeth have been set up so
that the disto-buccal cusp of the mandibular
first molar rests in the central fossa of the
maxillary first molar. Which of the following
BEST describes this occlusal classification?
A.
B.
C.
D.
Class I
Class II, division I
Class II, division II
Class III
Test item #'5 390-400 refer to the following testlet.
A patient with hepatitis C is scheduled for extraction of
posterior teeth in the right lower quadrant due to
advanced periodontal disease. Tooth #28 is
abscessed and draining into medial soft tissue. Tooth
#30 shows a large carious lesion and is painful.
390. Infection with hepatitis C results in liver
dysfunction, which might decrease the liver's
ability to store glycogen. In addition to the
liver, which of the following is the other major
site of glycogen storage?
A. Brain
B. Kidney
C. Smooth muscle
D. Cardiac muscle
E. Skeletal muscle
391. Damage to the liver from hepatitis C infection
might impair bilirubin metabolism. Bilirubin is a
product of which of the following pathways?
392.
A. Albumin synthesis
B. Cholesterol synthesis
C. Heme degradation
D. Copper degradation
-
Death from advanced liver disease caused by
hepatitis C infection is PRIMARIL Y due to
which of the following?
A. Inhibition of replication
B. Inhibition of transcription
C. Inhibition of urea synthesis
D. Increase in serum glucose levels
E. Inhibition of immunoglobulin production
393. What is the MOST likely outcome of this
patient's hepatitis C?
A. Development of chronic hepatitis
B. Development of fulminant hepatitis
C. Development of hepatocellular
D.
E.
carcinoma
Asymptomatic carrier state, with
normal serum transaminase
Resolution, with eventual clearing of
the virus
2005 American Dental Association, Joint Commission on National Dental Examinations. All rights reserved. Reprinted by permission.
41
42
394. If the patient transmits the hepatitis C virus to
the dentist, then it will MOST likely be via
which of the following routes?
395.
A.
B.
C.
D.
Oral-fecal
Saliva contamination
Inhalation of aerosols
Contaminated needle stick
Microbial analysis reveals that the flora of the
abscess of Tooth #28 consists predominantly
of P. gingiva/is. The inflammation is caused
primarily by which component of this
microorganism?
A. Exotoxin
B. M-protein
C. Lipoteichoic acid
D. Lipopolysaccharide
396. Which of the following is a possible life-
threatening sequela of the abscess of Tooth
#28?
A.
B.
C.
D.
E.
Allodynia
Hyperalgesia
Non-vital pulp
Referred pain
Ludwig's angina
397. Which of the following nerve fibers is primarily
responsible for the sharp pain in Tooth #30?
A.
B.
C.
D.
E.
Aa
AI3
A'6
B
C
398. If the patient decides to attempt to save Tooth
#30, then it will require root canal therapy.
Which of the following best describes the
MOST likely presentation of pulp canals in this
tooth?
A.
B
c
Two canals, one in each root
Three canals, two in the mesial root
and one in the distal root
Three canals, one in the mesial 1'00t
and two in the distal root
D. Four canals, two in each root
399. If the patient and dentist elect to extract Tooth
#30, sectioning of the tooth will be required.
The dentist should be aware of which of, the
following furcations on this tooth?
400.
A.
B.
C
D.
E.
One
Two - mesial and distal
Two - facial and lingual
Three - mesial, distal, and facial
Three - facial, mesial, and lingual
If the mandibular right posterior teeth are
extracted, then the maxillary right posterior
teeth will likely show signs of
A. attrition.
B. abfraction.
C. ankylosis.
D. mesial drift.
E. supraeruption.
WHEN YOU HAVE F!NISHED THIS
EXAMINATION BOOKLET, PLEASE
COMPLETE THE BRIEF CANDIDATE'S
QUESTIONNAIRE.
WHEN YOU HAVE COMPLETED THE
QUESTIONNAIRE, TURN IN THE
FOLLOWING MATERIALS TO THE TEST
ADMINISTRATOR:
1. THE TWO ORIGINAL (TOP) ANSWER
SHEETS
2. THE CANDIDATE QUESTIONNAIRE
THE TEST ADMINISTRATOR WILL PROVIDE
YOU WITH THE ANSWER KEY FOR THE
ENTIRE COMPREHENSIVE PART I PILOT
EXAMINATION.
YOU MAY KEEP THE EXAMINATION
BOOKLET AND THE CANDIDATE'S copy
OF THE TWO ANSWER SHEETS.
THANK YOU FOR YOUR PARTICIPATION IN
THIS PROJECT.
2005 American Dental Association, Joint Commission on National Dental Examinations. All rights reserved. Reprinted by permission.
3
Item No.
~
201
202
203
204
205
206
207
208
209
210
211
212
213
214
215
216
217
218
219
220
221
222
223
224
225
226
227
228
229
230
231
232
233
234
235
236
237
238
239
240
241
242
243
244
245
246
247
248
249
250
NATIONAl SOARD COMPREHENSIVE PART 8 P!LOT EXAMSNATION
TEST KEY
Discipline Item Key IwmNa. Discipline
Anatomic Sciences B 251 Biochemistry-Physiology
Biochemistry-Physiology C 252 Microbiologv-Pathology
Microbiology-Patholocy D 253 Dental Anatomy & Occlusion
Dental Anatomy & Occlusion C 254 Anatomic Sciences
Biochemistry-Physiology D 255 Microbioloav-Patholooy
Dental Anatomy & Occlusion
A
I
256 Biochemistrv-Phvsiology
Microbiolooy-Pathology D
I
257 Dental Anatomy & Occlusion
Anatomic Sciences
C 258 Anatomic Sciences
Biochemistry-Physiology E 259 BiochemistrY-Physiology
Microbiology-Pathology C 260 BiochemistryPhysiology
Dental Anatomy & Occlusion B 261 MicrobioloQv-PatholoQY
Microbiology-Pathology A
Biochemistry-Physiology D
262 Microbiology-Pathology
263 Biochemistrv-PhysiOiOQy
Anatomic Sciences C 264 Biochemislry-PhVSioiogy
Dental Anatomy & Occlusion B 265 Dental Anatomy & Occlusion
Dental Anatomy & Occlusion A 266 Denial Anatomv & Occlusion
MicrobioJogy-Patho!OfJ'J E 257 Anatomic Sciences
Biochemistry-Physiology D 268 Microbiology-Pathology
Anatomic Sciences C 269 Dental Anatomy & Occlusion
Dental Anatomy & Occlusion B 270 Dental Anatomv & Occlusion
Dental Anatomy & Occlusion 0 271 Microbiology-Pathology
Microbiology-Patholoav 0 272 Anatomic Sciences
Biochemistry-Physiology C 273 Biochemistry-Physiology
Anatomic Sciences B 274 Microbioloav-Pathology
Biochemistry-Physiology B 275 Dental Anatomv & Occlusion
Anatomic Sciences B 276 Anatomic Sciences
Dental Anatomy & Occlusion C 277 Biochemistry.-PhysioJQqV
Biochemistry-Physiology D 278 Microbiology-Pathology
Microbiology-Pathology C 279 Dental Anatomy & Oc,clusion
Anatomic Sciences A 280 Microbiolowt-Patholomr
Dental Anatomy & Occlusion 0 281 Biochemistry-Physiology
Microbiology-Pathology D 282 Anatomic Sciences
Biochemistry-Physiology C 283 Biochemistry-Physiology
Anatomic Sciences 8 284 Anatomic Sciences
Biochemistry-Physiology C 285 Dental Anatomy & Occlusion
Dental Anatomy & Occlusion D 286 Microbiology-Pathology
Microbiology-Pathology A 287 Biochemistry-Physiology
Anatomic Sciences E 288 Anatomic Sciences
Anatomic Sciences A 289 Dental Anatomv & Occlusion
Biochemistry-Physiology D 290 Microbiology-Patholoov
Microbioloav-Patholoov B 291 Biochemistry-Physiology
Dental Anatomy & Occlusion C 292 Anatomic Sciences
Anatomic Sciences C 293 Biochem istry-PhVSioloov
Microbiology-Pathology E 294 Microbiology-Pathology
Dental Anatomy &. Occlusion C 295 Dental Anatomy & Occlusion
Anatomic Sciences B 296 Microbiology-Pathology
MicrobioloQY-Pathology C 297 Biochemistry-Phvsioloav
Biochemistry-Physiology A 298 Anatomic Sciences
Anatomic Sciences A 299 Anatomic Sciences
Dental Anatomy & Occlusion B 300 Biochemistry-Physiology
Hem Key
C
B
A
E
A
E
A
D
B
D
D
E
C
D
C
D
E
C
E
D
--
C
A
B
D
B
C
B
B
A
D
B
E
A
B
A
C
D
B
A
B
A
A
D
E
A
B
E
B
D
C
@ 2005 American Dental Association, Joint Commission on National Dental Examinations. All rights reserved. Reprinted by permisSion.
Item No.
""'"
301
302
303
304
305
306
307
308
309
310
311
312
313
314
315
316
317
318
319
320
321
322
323
324
325
326
327
328
329
330
331
332
333
334
335
336
337
338
339
340
341
342
343
344-
345
346
347
348
349
350
NA,'T20NAL 80ARD COMPREHEf'JS5VE PART I PILOT EXAMINATION
TEST KEY
Discipline Item Key Item No. Discipline
Microbiology-Pathology A
I
351 ,'-','""-" emistry-Physiology
Dental Anatomy & Occlusion C 352 Dental Anatomy & Occlusion
Anatomic Sciences A 353 Dental Anatomy & Occlusion
Biochemistry-Physiology C 354 Anatomic Sciences
Microbiology-Pathology A
,
355 Biochemistry-Physiology
Dental Anatomy & Occlusion A 356 Microbiolomr-Pathology
Biochemistry-Physiology E 357 Dental Anatomy & Occlusion
Anatomic Sciences E 358 Anatomic Sciences
Microbiology-Pathology E 359 Biochemistry-Physiology
Dental Anatomy & Occlusion A 360 Microbiology-Pathology
Dental Anatomy & Occlusion C 361 Dental Anatomy & Occlusion
Microbiology-Pathology C 362 Microbiology-Pathology
Biochemistry-Physiology A 363 Biochemistry-Physiology
Anatomic Sciences D 364 Biochemistry-Physiology
Biochemistry-Physiology C Anatomic Sciences
Microbiology-Pathology E 66 Anatomic Sciences
Dental Anatomy & Occlusion D 367 ,Anatomic Sciences
Dental Anatomy & Occlusion A 368 Dental Anatomy & Occlusion
Microbiology-Pathology E 369 Microbiology-PathoJogy
Biochemistry-Physiology B 370 Microbiology-Pathology
Anatomic Sciences B 371 Microbiology-Pathology
Biochemistry-Physiology C 372 Biochemistry-Physiology
MicrobiologyPatholClY C 373 Anatomic Sciences
Dental Anatomy & Occlusion B 374 Dental Anatomy & Occlusion
Microbiology-Pathology B 375 Dental Anatomy & Occlusion
Biochemistry-Physiology 0 376 Dental Anatomy & Occlusion
Anatomic Sciences E 3n Anatomic Sciences
Microbiology-Pathology C 378 Microbiology-Pathology
Dental Anatomy & Occlusion A 379 Microbiology-Pathology
Dental Anatomy_ & Occlusion A 380 Anatomic Sciences
Microbiology-Pathology C 381 Microbiology-Pathology
Anatomic Sciences E 382 Mlcrobiology-PathoJogy
Biochemistry-Physiology B 383 Biochemistry-Physiology
MicrobiologyPathology B 384 Biochemistry-Physiology
Dental Anatomy & Occlusion C 385 Biochemistry-Physiology
Microbiology-Pathology E 38S Dental Anatomy & Occlusion
Biochemistry-Physiology B 387 Anatomic Sciences
Anatomic Sciences C 388 Anatomic Sciences
Mlcroblology-Pathole>gy B 389 Dental Anatomy & Occlusion
Dental Anatomy & Occlusion E 390 8iochemistry-Physioloav
Anatomic Sciences D 391 Biochemistry-Physiology
Microbiology-Pathology E 392 Biochemistry-Physiology
Dental Anatomy & Occlusion E 393 Microbiology-Pathology
Anatomic Sciences 8 394 Microbiology-Pathology
Dental Anatomy & Occlusion B 395
Microbiology-Pathology
Microbiology-Pathology D 396 Anatomic Sciences
Biochemistry-Physiology C 397 Anatomic Sciences
Anatomic Sciences D 398 Dental Anatomy & Occlusion
Dental Anatomy & Occlusion D 399 Dental Anatomy & Occlusion
Microbiology-Pathology E 400 Dental Anatomy & Occlusion
Item Key
D
E
0
C
A
B
A
C
B
E
B
C
D
D
B
C
A
E
A
C
D
D
E
C
C
C
E
C
E
B
A
E
A
E
A
E
C
8
A
E
C
C
A
D
D
E
C
8
C
E
2005 American Dental AssOCiation, Joint Commission on National Dental Examinations. All rights reserved. Reprinted by permission.
4
National Board DentaJ
TEST PACKET I-M
Compiled, released items from approximately 1998-2005
American Student Dental Association
211 hlsl ( Avenue' Chic:l!2(l, Jllinoi, (,0(, II I ;>440279:')
I 1\1;111: J\SI)A(u'AS! Weh,;I" wwwASIJ/\J\l'Lm;:
2]] Eas1 Chicago Avenue Chicago, Illinois 606] J 3]2-440-2795 Fax: 3]2-440-2820
Contents
E-Mail: ASDA@ASDAne1.org Website: www.ASDAne1.org
TEST PACKET I-M
National Board Dental Examination Part I
(Compiled, released items from
approximately 1998-2005)
Page
Randomly ordered test items ........................................................... 3
Answer Key .................................................................................... 22
2.
3.
Which of the following nerves innervates the
Inferior buccal mucose of the flom of the oral
cavity?
A. Facial
B. Trigeminal
C Lingual
D. Mylohyoid
E. Hypoglossal
Each of the following structures increases the
suliace area of the small intestine EXCEPT one.
Which one is the EXCEPTION?
A Villi
B. Rugae
C. Microvilli
D. Brush border
E. Circular folds
The inferior parathyroid gland develops from which
of the following structures?
A
B
C.
D.
E.
Second pharyngeal arch
Fourth pharyngeal arch
Fifth pharyngeal arch
Third pharyngeal pouch
Fourth pharyngeal pouch
4. Which of the following controls parathyroid gland
function?
A
B.
C.
D.
E.
Thyroxin
Calcitonin
Blood levels of calcium
Blood levels of magnesium
Thyroid stimulating hormone
5. The splenic artery is a branch of which of the
following arteries?
A
B.
C.
D.
E.
Celiac
Left hepatic
Left gastric
Musculophrenic
Superior mesenteric
3
6
7.
8
9.
10.
Which of the following muscles partici[Jates in
flexion at tile gleno-humeral and the humero-ulnar
joints?
A Deltoid
B. Blachialis
C Biceps brachii
D. Triceps brachii
E. Coracobrachialis
Cell bodies of sympathetic fibers in the nerve of
the pterygOid canal come from which of the
following?
A Facial nerve
B. Superior cervical ganglion
C. Greater petrosal nerve
D. Glossopharyngeal nerve
E. Otic ganglion
In the upper limb, which of the following represents
a hallmark of lymphatic vessels?
A
B.
C.
D.
Contain valves
Follow the veins
Always travel in pairs
Only found on the anterior suliace of the
limb
E. Contain fenestrations to allow passage of
fluids into the interstitium
Which of the following nerves is associated wittl
the second pharyngeal arch?
A Vagus
B. Glossopharyngeal
C. Accessory
D. Mandibular
E. Facial
The muscles of facial expression are derived from
which of the following?
A Frontonasal process
B. First arch
C. Second arch
D. Third arch
E. Fifth al'ch
11 Each of the following crclilial nerves is associated
with the cavernous sinus EXCEPT one. Which one
is the EXCEPTION?
A. Facial
B. Abducens
C. Trochlear
D. TI'igeminal
E. Oculomotor
12. Which of the following nerves innervates the taste
buds of the anterior two-thirds of the tongue?
A.
B.
C.
D.
E.
Vagus
Facial
Trigeminal
Hypoglossal
Glossopharyngeal
13. A deviation of the tongue, when protruded, away
from the midline results from damage to which of
the following cranial nerves?
A.
B.
C.
D.
E.
V
VII
X
XI
XII
14. Which of the following exhibits phagocytic activity
in the central nervous system?
15.
A. Ependymal cell
B. Microglial cell
C. Oligodendrocyte
D. Fibrous astrocyte
E. Protoplasmic astrocyte
Each of the following cell layers is derived from
oral epithelium EXCEPT one. Which one is the
EXCEPTION?
A.
B.
C
D.
E.
Dental follicle
Stellate reticulum
Stratum intermedium
Inner enamel epithelium
Outer enamel epithelium
4
16. The nerves of the internal thoracic wall lire
immediately deep to which layer of the wal!?
A. External intercostal
B. Internal intercostal
C. Transversus thoracis
D. Parietal pleura
E. Subcostals
17. Which of the following represents the condition in
which a radiograph of a 10-year-old patient's
maxilla shows that two succedaneous teeth are
absent?
A.
B.
C.
D.
E.
Mesiodens
Gemination
Dens in dente
Latent odontogenesis
Partial anodontia (hypodontia)
18. Where is the channel of communication between
the maxillary sinus and the nasal cavity situated?
19.
A.
B.
C
D.
In the superior nasal meatus
In the sphenoethmoidal recess
In the middle nasal meatus, at the semilunar
hiatus
At the anterior extremity of the inferior nasal
meatus
E. At the posterior extremity of the inferior nasal
meatus
In the temporomandibular joint, a very dense
collection of organized elastic fibers is found in
which of the following areas of the articular disc?
A. Anterior band
B. Posterior band
C. Intermediate band
D. Posterior-inferior lamina of the bilaminar
zone
E. Posterior-superior lamina of the bilaminar
zone
20. Which of the following represents a muscle that
might assist in depressing tile mandible?
A. Masseter
B. Stylohyoid
C. Teillporalis
D. Geniohyoid
E. Medial pterygoid
21. The organic componenl of dentin is comprised
primarily of
/\.
B.
C
o
E.
keralin.
reticular fibers.
type I collagen.
tyre II and III collagell.
oxytalan fibers.
22 Each of the following structures leaves the cranium
by way of the jugular foramen EXCEPT one. Which
one is the EXCEPTION?
A
B.
C
o.
Cranial nerve IX
Cranial Ilerve X
Cranial nerve XI
Cranial nerve XII
E. Sigmoid sinus - internal jugular vein
23. Each of the following venous channels has direct
connections with the pterygoid venous plexus
EXCEPT one. Which one is the EXCEPTION?
24.
A MaXillary vein
B. Vertebral vein
C. Deep facial vein
D. Infraorbital vein
E. Posterior superior alveolar vein
Stimulation of the lesser petrosal nerve in an adult
causes secretion by which of the following glands?
A
B.
C
o.
E.
Parotid
Lacrimal
Sublingual
Submandibular
Glands of the hard palate
25. Ameloblasts will not differentiate from
preameloblasts unless they
A
B.
C.
D.
E.
contact dentin
are touched by odontoblast processes
are touched by stratum intermedium
contact stellate reticulum
contact neural crest mesoderm
5
26. Cells thai will form the vertebrae have their origin
in which of the following?
A.
B.
C
D.
E.
Notochord
Neural arch
Dermamyotome
Two pairs of somites
Intermediate mesodermal plate
27. The nasolacrimal duct drains into which of the
following?
A
B.
C
D.
Middle meatus
Inferior meatus
Superior meatus
Sphenoethmoidal recess
28. The carotid sheath encloses each of the following
structures EXCEPT one. Which one is the
EXCEPTION?
29.
A. Vagus nerve
B. Extemal carotid
C. Internal jugular vein
D. Common carotid artery
Following eruption and initial occlusal contact in
the oral cavity, a tooth will continue to erupt in
order to compensate for occlusal wear. In response
to this continuous eruption, which of the following
is deposited at the apex of the root?
A.
B.
C.
D.
E.
Primary dentin
Secondary dentin
Reparative dentin
Cellular cementum
Acellular cementum
30. Which of the following cells is most likely to be
engaged in mitosis at any given time?
A
B.
C
D.
E.
Osteocyte
Macrophage
Plasma cell
Chondrocyte
Basal keratinocyte
31. Fibrocanilage normally occurs in
A.
8.
C.
o
E.
epiglottis
external ear.
epiphyseal plate.
inter'venebral discs.
C-shaped rings ill the wall of the trachea
32. There is a distinct change in the type of surface
epithelium at the junction of the
A
B.
C
D.
E.
oropharynx and esophagus.
esophagus and stomach.
fundus and pylorus.
duodenum and jejunum.
ileum and colon.
33. In which of the following would fenestrated
capillary plexi be found?
A
B.
C.
D.
E.
Predentin
Dental pulp
Cellular cementum
Periodontal ligament
Reduced enamel epithelium
34. Submucosal glands are usually located in the
A
B.
C.
D.
E.
fundus of stomach.
duodenum
appendix.
jejunum.
colon.
35. Difficulty in raising the shoulder might be the result
of damage to which of the following cranial
nerves?
A. V
B. VII
C. X
O. XI
E. XII
36. A stab wound creating a pneumothorax on the left
side will usually result in collapse of which of the
following?
A The left lung only
B. The right lung only
C. Both lungs
D. The rib cage on the left side
E The lef1lung and pericardial sac
37. Shar'pey's fibers from the periodontal ligament
ins(Ori into which of the following structures?
A
B.
C.
D.
E.
Bundle bone and dentin
Conical plates and dentin
Bundle bone and cementum
Conical plates and cementum
Haversian bone and cenientum
38. In an adult, a Babinski sign indicates damage to
A
B.
C.
D.
E.
the dorsal horn.
lower motor neurons.
upper motor neurons.
the lateral spinothalamic tract.
the dorsal spinocerebellar tract.
39. Cell bodies of neurons mediating proprioception
from the periodontal ligament of the maxillary first
molar lie primarily in the
6
40.
A
B.
C.
D.
E.
semilunar ganglion.
motor nucleus of V.
spinal nucleus of V.
chief sensory nucleus of V
mesencephalic nucleus of V
In hydrocephalus, excess cerebral spinal fluid is
found within which of the following?
A. Ventricle
B. Subdural space
C. Cisterna magna
D. Subarachnoid space
E. Superior sagittal sinus
41. Intelligence and sensory-motor functions of a
patient appear to be intact. However, the patient
lacks self-discipline and has exhibited anti-social
behavior. He is unable to plan for the future or to
organize behaviors into logical sequences. A lesion
in which of the following areas is likely?
A.
B.
C.
D.
E.
Frontal lobe
Occipital lobe
Internal capsule
Teillporallobe
Parietal lobe
42. The neurons of the central nervous system that
innervate muscles derived from branchial arches
are found In which of the following nucle()
A
B
C
o
E.
Nucleus ambiguus
Hypoglossal nucleus
Dorsal motor nucleus of X
Nucleus of Edinger-Westphal
Superior salivatory nucleus
43. Which of the following represents the basis for the
topical application of fluorides in dental caries
pr'evention?
A. Fluoride penetrates the enamel through the
lamellae.
B Keratin content of the enamel is made more
insoluble.
c
D.
E.
Fluoride coats the tooth forming a uniform
protective barrier.
The primary cuticle, being less calcified,
absorbs the fluoride.
Acid solubility of the surface enamel is
reduced by the fluoride.
44. Which of the following represents the primary
mineral component of alveolar bone in the
periodontium?
A Osteoid
B
C.
D.
E.
Elastin
Collagen
Hydroxyapatite
Ground substance
45. Which of the following represents the primary
function of cementum?
A
B.
C
D
E.
Attach Sharpey's fibers
Protect the root from caries
Repair traumatic injuries of the root
Supply nutrition to the periodontal ligament
Maintain the width of the periodontal
ligament
46. The junction between the tooth surface and the
crevicular epithelium is composed of whiCh of the
following?
A Cementoid
B. Tight junction
C Intermediate filaments
D. Basal lamina-like structure
E. Interstitial crevicular fluid
7
47. Mucosa of the anterior two-thirds of the tongue
develops primaril)' from
A.
B.
C
D.
E.
Rathke's pouch
tuberculum impar.
foregut endoderm.
hypobranchial eminence.
lateral lingual swellings.
48. Initial/y, the developing hean is
49.
A.
B.
C
D.
induced by the notochord.
positioned posterior to the notochord.
positioned anterior to the prochordal plate.
between the prochordal plate and the
notochord.
Melanocytes migrate to the lamina propria of the
oral mucosa from which of the following?
A Myotome
B. Dermatome
C Sclerotome
D. Neural crest
E. Rathke's pouch
50. Blood from the cephalic vein drains into which of
the following veins?
A
B.
C.
D.
E.
Basilic
Braciocephalic
Axillary
Internal thoracic
Superior vena cava
51. Coupled respiration requires each of the following
EXCEPT one. Which one is the EXCEPTION?
A
B.
C.
D.
E.
ADP
Oxygen
Carbon dioxide
Electron donor
Inorganic phosphate
52. If the anticodon on transfer-RNA is 5'ACG3', then
which of the following is its corresponding codon
on messenger-RNA?
A 5' CGT 3'
B. 5' CGU 3'
C. 5' TGC 3'
D 5' UAG 3'
E. 5' UGC 3'
Which of the following is the major storage form of
iloll in the-;
A.
B.
C
D.
E
Transferrin
Hemosiderin
Apoferritin
Hemoglobin
Ferrilin
5-'1. When a mammalian red blood cell is placed in 0.3
per cent sodium chloride, water moves across the
cell membrane by which of the following
processes?
A
B.
C
D.
E.
Osmosis
Hemolysis
Pinocytosis
Active transport
Diffusion
55. Free fatty acids in plasma are
A.
B.
C.
D.
metabolically inert.
mainly bound to [beta]-lipoproteins.
independent of epinephrine secretion.
mainly associated with serum albumin.
56. Each of the following is required for normal blood
Glot formation EXCEPT one. Which one is the
EXCEPTION?
A Ca++
B.
Plasmin
C.
Thrombin
D.
Vitamin K
E.
Proteolysis
57. Which of the fOllowing combines with heparin to
inhibit blood clotting?
A.
B.
C
D.
E.
Anti-thrombin III
Platelets
Fibrinogen
Plasminogen
Thrombin
8
5A Each of the following is an anticoagulant EXCEPT
one. Which one is the
/J,.
B.
C.
D.
E.
Heparin
Vitamin K
Dicumarol
Sodium citrate
Any antithrombin sUbstance
59 The characteristic of the aorta lhat is most
responsible for the maintenance of diastolic blood
pressure is its
A
B.
C.
D.
E.
wide lumen.
elastic distenSibility.
proximity to the heart.
great peripheral resistance.
active contraction.
60. Colloid osmotic pressure of the blood is important
because it
A
B.
C.
D.
E.
nourishes blood cells.
aids in blood clotting.
prevents loss of erythrocytes from the blOod.
prevents excess loss of fluid from capillaries.
prevents entrance of tissue fluid into
capillaries.
61. Which of the following is most likely to promote
de polymerization of extracellular matrix?
A. Cortisone
B. Collagenase
C. Chymotrypsin
D. Hyaluronidase
62. Which portion of the nervous system contains the
cardiac, vomiting, and vasomotor centers?
A
B.
C
D.
Medulla
Thalamus
Cerebrum
Cervical region of the spinal cord
64
The concentration of which of the following amino
acids can be used to determine whether or not
dentin has contaminated a purified enaillel
preparation?
A.
B.
C.
o.
E.
Lysine
Proline
Hydroxyproline
Phosphotyrosine
Phosphoserine
Calcium is transported in the blood in combination
with which of the following?
A. AlbUmin
B. Citrate
C. Heillogiobin
D. Beta-globulin
65. Sympathetic stimUlation most likely produces which
of the following?
A.
B.
C.
D.
E.
Glycogen synthesis
Bronchial dilation
Decreased mental activity
Decreased metabolic rate
Increased blood flow to the skin
66. Which of the following is the same value for
intracellular and interstitial fluid?
A. [Na+]
B. [Cn
c. [K+]
D. Colloid osmotic pressure
E. Total osmotic pressure
67. Which of the following explains why an increased
filling of the ventricle during diastole causes a
more forceful heali beat? The increased ventricular
volume
A. dimin'lshes the refractory period of the
ventricle.
B. increases end-diastolic fiber length.
C. facilitates conduction in the heart.
D. produces a less than optimal load.
9
68. Prior to surgery an anxious patient has a higher
systoliC blood pressure than previously noted.
Which of the following represents the most likely
reason';
A.
B.
C.
D.
E.
Cardiac shock
Anaphylactic shock
Atrial fibrillation
Ventricular fibrillation
Decreased arterial compliance
69. Reduced renal blood flow can cause hypertenSion
by which of the following?
A. Release of renin
B. Reflex vasoconstriction
C. Retention of potassium in blood
D. Increased output of epinephrine
70. Which of the following describes the effect of a
drug that inhibits renal carbonic anhydrase?
A.
B.
C.
O.
It decreases urea clearance.
It increases the Tm for glucose.
It increases the acidity of the urine.
It decreases sodium reabsorption in the
proximal tubule.
71. The composition of plaque is most similar to which
of the following?
A. Bone
B. Enamel
C. Calculus
D. Oral bacteria
E. Saliva
72. Which of the following substances is the
predominant source of ATP at MODERATE levels
(for greater than 60 minutes) of activity?
A.
B.
C.
O.
Amino acids
Fatty acids
Carbohydrates
Proteins
73
74
75.
Which of the following substances has a ciearance
rate that is greatel than the glomerular filtration
rate (GFRfJ
A. Para-aminohippuric acid (PAH)
B Potassium
C Glucose
D. Urea
What is the biological Significance of the extensive
degeneracy of the genetic code?
A
B.
Alters the amino acid sequence of proteins
encoded by the DNA
Minimizes the deleterious effect of mutations
C. Maximizes the beneficial effect of mutations
O. Increases chain termination
E Leads to active proteins
Which of the following types of blotting can be
used to identify DNA restriction fragments?
A.
B.
C.
O.
Eastern
Southern
Northern
Western
76. The catabolism of which of the following results in
no energy production in the form of ATP?
A
B.
C.
O.
Lipid
Protein
Nucleotide
Carbohydrate
77. Which of the following substances is released by
blood platelets and causes platelets to stick
together?
A.
B.
C
O.
E.
Thrombin
Fibrinogen
Phospholipids
Throillboxane A2
Prostacyclin 12 (PgI2)
10
78. Peptide hormones generally exert their effect
thlOugh which of the iollowlllg'i
A.
B.
C.
O.
Intracellular receptms and stimulating protein
synthesis
Receptol's on the cell membrane and
stimulating protein synthesis
Intracellular receptors and altering the
specific activity of certain enzymes
Receptors on the cell membrane and
altering the specific activity of ceriain
enzymes
79. During the period of isovolumetric contraction,
which of the following happens?
A
B.
C.
o
The semilunar valves are open.
The left ventricular pressure is rising rapidly.
The aortic pressure is slightly less than the
left ventricular pressure.
The right ventricular pressure is greater than
the left ventricular pressure.
80. Glucose reabsorption in the nephron is usually
completed in which of the following?
A.
B.
C.
O.
Distal tubule
Loop of Henle
Collecting duct
Proximal tubule
81. Which of the following represents the correct size
and characteristic of the nerve fibers that conduct
sensory input of pain from the oral-facial region?
A
B.
C.
D.
E.
Large diameter, myelinated
Small diameter, myelinated
Large diameter, unmyelinated
Small diameter, unmyelinated
Intermediate diameter, myelinated
82. As blood flows through the periodontal melllbrane.
t"emendous pressures occur in response to
forceful occlusion. Blood flow is teillporar-ily
reduced, but platelets do not aggregate because of
the presence of
A. Ca++
B. AOP.
C. thrombin.
O. thrornboxane A2.
E. prostacyclin (PgI2).
8:1. /\n examp\(" 01 synergism ic; the efiectc, 01
C
o
insulin and glucagon on blood glucose
estrogen and progester'one on uterine
motilrty.
growth hormone and thyroxine on skeletal
growth
antidiuretic hormone and aldoster'one on
potassiulll exuetion
84. The jaw jerk reflex is an example of which of the
following reflexes?
A Load
B. Flexor
C. Withdrawal
D. Dynamic stretch
85. The stretch reflex is an example of which of the
fol/owing reflexes?
86.
87.
A Withdrawal
B. Nociceptive
C. Polysynaptic
D. Monosynaptic
A patient bites down rapidly on an unexpected
hard surface while chewing. Cessation of motor
unit recruitment in jaw closing muscles is caused
by stimulation of
A
8.
C.
D.
muscle spindles.
mucosal mechanoreceptors.
periodontal mechanoreceptors.
nociceptors in the dental pulp.
Which of the following statements about
norepinephrine is carTee!?
A Causes cardiac acceleration
8. Causes general vasodilation
C. Causes vasodilation in vessels of the skin
D.
E
Has a negative inotropic effect on the heart
Is the preganglionic sympathetic
neu rotr'ansmitter
11
88. Which of the following molecules causes the
activatiorl of myosin kinase and the contraction of
smooth muscle'l
A.
B.
C.
O.
E.
Actin
Troponin
Calmodulin
Calcitonin
Cholecalcileml
89. Which of the following is the primary difference
between juxtamedullary and cortical nephrons?
90.
A Renal plasma flow
B. Filtration fraction
C. Length of the distal tubule
D. Length of the proximal tubule
E. Length of the thin segment of the loop of
Henle
Which of the following is an essential element
found in all cytochromes?
A. Co
B. Cu
C Fe
D. Mg
E. Zn
91. Acute cyanide poisoning would most likely lead to
which of the following?
A
B.
c.
D.
E.
Increased oxygen concentration in arterial
blood
Increased carbon dioxide concentration in
venous blood
Decreased oxygen extraction by peripheral
tissues
Increased oxygen extraction by peripheral
tissues
Decreased carbon dioxide concentration in
arierial blood
92. Chondroitin sulfate is a major component of which
of the following?
A. Hair
B. Mucin
C. Cartilage
D. Bacterial ceil walls
E. Blood group substance
93. Which of it-Ie following characterizes hoth activ("
transpori and facilitated diffusion?
A. Hydrolysis of ATP
B. Competitive inhibition
C Transport bidil-ectionai
D Transport against a concentration gradient
94 _ Which of the following has the most effect in
stimulating respiration?
A. Increase in blood pH
B. Decrease in arterial p0 2
C.
D.
E.
Increase in arte(lal pC02
Decrease in blood pH
Decrease 'In arterial pC0
2
95_ Which of the following proteins is involved with
bacte(lal aggregation and subsequent eliminat'lon
from the oral cavity?
A_
S.
C.
D.
Gustin
Statherin
Lactoferrin
Secretory IgA
96. Thyroid hormones are synthesized from which of
the following amino acids?
97.
98_
A. Tyrosine
B. Dopamine
C. Histidine
D. Threonine
E. Tryptophan
Which of the following enzymes is responsible for
inactivating catecholamines?
A. Phosphodiesterase
B. Monoamine oxidase
C. Amino decarboxylase
D. Tyrosine hydroxylase
Protein and RNA synthesis occur in each of the
following phases of the cell cycle EXCEPT one.
Wilich one is the EXCEPTION?
A. GO
B.
G1
C. S
D.
G2
E. M
99. Whicll main product of protein nitrogen metabolislTI
is 10und in human urine?
A Urea
B. Ammonia
C. Creatine
D. Uric acid
E Creatinine
100. Each of the following is expected to be active
during fatty acid biosynthesis EXCEPT one. Which
one is the EXCEPTION?
A.
B.
C.
D.
E.
Tricarboxylic acid cycle
Pyruvate dehydrogenase
Amino acid catabolism
Beta oxidation
Glycolysis
101. Which of the following statements best describes
the roots of a maxillary first molar?
12
A.
B.
C.
The mesiofacial root usually tips mesially in
its apical one-third.
The palatal roo\ has a concave facial
surface.
The palatal root is flattened and concave on
its mesial and distal surfaces.
D. The distofacial root is flattened and concave
on its facial surface.
102. Which of the following represents the location of
the lingual height of contour on the crown of the
mandibular second premolar?
103.
A. Middle third
B. Occlusal third
C. Same third as that tooth's buccal height of
D.
E.
contour
Same third as the lingual height of contour
on the crown of the maxillary premolars
Same third as the lingual height of contour
on the crown of the mandibular first premolar
When v'lewed from the facial aspect, the crown of
the mandibular first premolar has an occlusal
outline that normally exhibits which of the following
characteristics?
A. Cusp tip which is offset to the distal
B. Cusp tip which is centered mesiodistally
C. Disto-occlusal slope which is longer than the
mesio- occlusal slope
D _ Mesio-occlusal slope which is longer than
the disto- occlusal slope
~ 04. Two pulp canals are most commonly found in the
A.
B.
C
D
E
root of a mandibular central incisor.
facial root of a maxillary first premolar
distal root of a mandibular first molar.
mesial root of a mandibular first molar.
mesiofacial root of a maxillary first molar.
105. In the triangle formed by the projection of the
orifices of the canals of a maxillary first molar, the
A.
B.
C
D
E.
line connecting mesial with lingual is longest.
line connecting distal with lingual is longest.
line connecting mesial with distal is longest
angle at the distolingual canal is obtuse.
angle at the mesiofacial canal is obtuse.
106. Which of the following attaches the root surface to
the alveolar bone?
A. Cementum
B. Attached gingiva
C. Dentinal tubules
D. Periodontal ligament
E. Cementoenamel junction
107. Which of the following anatomic structures is found
just lingual to the maxillary central incisors?
A. Incisive foramen
B. Canine eminence
C. Maxillary tuberosity
D. External oblique ridge
E. Greater palatine foramen
10S. Which of the following papillae would normally be
found in the buccal vestibule?
A. Parotid
B. Incisive
C Fungiform
D. Interdental
E. Circumvallate
109. The bulk of a tooth consists of
A. pulp.
B. crown.
C. dentin.
D. enamel.
E. cementum.
13
110 The part of the tooth which, at a given moment, is
exposed to the oral cavity is
A.
B.
C.
D
E.
the anatomic crown.
the clinical crown.
the functional clinical crown and root.
measurably larger ill young per'sollS than in
older persons.
not efiected by periodontal health.
111. The anatomic crown is that portion of a tooth that
is
A.
B.
C.
D
in occlusion.
exposed to oral fluids.
coronal to the cervical line.
occlusal to the gingival margins.
112. A patient has an extremely wide, notched tooth in
the mandibular left central incisor position. Clinical
and radiographic examinations reveal 28 teeth
have erupted, but four third molars have not
erupted. Which of the following conditions exists?
A. Fusion
B. Dens in dente
C. Concrescence
D. Dilaceration
E. Gemination
113. When compared to a maxillary first molar, the roots
of a maxillary second molar
A. are longer.
B. are more divergent.
C. are fewer in number.
D. have less potential for fusion.
E. are greater in distal inclination.
114. Which of the following teeth is the most likely to
have a bifurcated root?
A.
B.
C.
D.
E.
Maxillary central incisor
Maxillary lateral incisor
Mandibular lateral incisor
Maxillary canine
Mandibular canine
115. Which of the following primary teeth does a second
premolar replace when it erupts into the oral
cavity?
A. Canine
B. First premolar
C. Second premolar
D. First molar
E. Second molar
11 (:, Which of the following describes tile contact
relationship between 8 I118xilialY central incisor allo
a maxillary lateral incisor?
A
B.
C
D.
Contact is offset to the lingual
Contact is centered incisocervically.
Lingual embrasure is larger than the facial
embrasure.
Incisal embrasure is the largest of all the
embrasures.
117. In the horizontal plane, as the mandible moves in a
lateral excursion, the midline of the mandible
moves
A.
B.
C.
D.
E.
straight anteriorly.
straight posteriorly.
straight laterally.
anteriorly and laterally.
posteriorly and laterally.
118. A patient presents with symptoms of an acute
abscess on the maxillary left lateral incisor. There
are no clinical signs of decay or restoration.
Radiographically, which of the following will most
likely be shown as the cause of the abscess?
A. Agenesis
3. Dilaceration
C. Concrescence
D. Dens in Dente
E. Enamel pearls
119. Each of the following can be found in the dental
pulp EXCEPT one. Which one is the EXCEPTION?
A.
B.
C.
D.
Nerve tissue
Blood vessels
Cementoblasts
Lymph vessels
120. Which of the following molar roots is wide
faciolingually and concave on both mesial and
distal surfaces?
A. Distofacial of a maxillary first
B. Lingual of a maxillary first
C Mesial of a mandibular first
D. Distal of a mandibular first
121, The distolingual cusp of which of the following
molal's might be absent?
A.
B.
C
D.
Maxillary first
Mandibular first
Mandibular third
Maxillary third
14
122. In a right working movement, the lingual cusp of a
maxillal'y rrght second premolar passes through
whicll of the following mandibular structures?
A.
B.
C
D.
FaCial groove of the right first molal
Lingual groove of the right first molar
Embrasure between the right first premolar
and the right second premolar
Embrasure between the right second
premolar and the right first molar
123, The proximal contact areas between anterior teeth
are incisal to the middle third of the teeth. Which 01
the following is an EXCEPTION to this rule?
A
B.
C.
D.
The mesial contact of a maxillary canine
The distal contact of a maxillary canine
The mesial contact of a maxillary lateral
incisor
The mesial contact of a mandibular lateral
incisor
124, When the mandible performs a laterotrusive
movement, the laterotrusive-side condyle moves
primarily about which of the following axes?
A. Vertical
B. Sagittal
C. Horizontal
D. Transverse
125. During a visual examination of a patient who has a
severe clenching habit, which muscle would one
expect to see enlarged?
A.
B.
C.
0,
Masseter
Temporalis
Medial pterygoid
Lateral Pterygoid
126. From the occlusal aspect, which of the following
represents the most frequently seen form of a
maxillary second molar?
A. Round
B. Square
C. Trapezoidal
D. Rhomboidal
E. Heal1-shaped
127. Initiation of calcification fm the mandibular central
incisor normally OCCUI'S at
A
B.
C.
D.
E.
3-4 months of age.
1 year of age.
2-3 years of age.
4-5 years of age.
6-7 years of age.
1 n The oblique ridge of the maxillalY molars extends
I)(-:twloell which two cusps?
A
B
C
D.
E.
Meslofacial and distofacial
Mesiolingual and distolingual
Mesiollllgual and distofacial
Mesiofacial and mesiolingual
Mesiofacial and distolingual
129. At age 8, the maxillary first molar has
A
B.
C
D.
E.
a mesial contact with the primary first molal.
a distal contact with the second molar.
no mesial contact.
no distal contact.
not yet erupted.
130. Of the primary maxillary teeth, the cervical ridge
would stand out most prominently as a distinct
entity on which surface of which molar?
Molar Surface
A First Distofacial
B. First Mesiofacial
C. Second Distofacial
D. Second Mesiolingual
E. Second Distolingual
131. Which groove of the mandibular first molar does
the maxillary mesiolingual cusp pass through in a
lateral excursive movement on the working side?
A Buccal
B. Lingual
C Central
D. Distobuccal
132. Which of the following premolars frequently has
only one pulp horn?
A
B.
C
D.
Maxillary first
Mandibular first
Maxillary second
Mandibular second
133. The illcisal embrasure is the smallest between
which of the following two teeth?
A. Maxillary central incisors
B Mandibular central incisol's
C Maxillary central and lateral incisors
D. Maxillary lateral incisor and canine
E Mandibular lateral incisor and central inciSal
15
134. When in its normal position relative to the arch
fornl thce: crown of a mondibular first 1110101 inclines.
A.
B.
C.
D.
distally and facially.
mesially and facially.
mesially and lingually
distally and lingually.
135. Which of the following teeth mighl possess three
cusps?
A
B.
c.
D.
E.
Maxillary second pl'emolar and maxillary first
molar
Maxillary second premolar and mandibular
first premolar
Maxillary first molar and mandibular second
molar
Maxillary second molar and mandibular first
premolar
Maxillary second molar and mandibular
second premolar
136. Each of the following grooves originates in the
central pit of the maxillary second molar (four cusp
type) EXCEPT one. Which one is the
EXCEPTION?
A.
B.
C.
D.
Buccal
Central
Distolingual
Transverse grooves of oblique ridge
137. When the mandible moves from maximum
intercuspai position distally, any tooth contacts that
occur are called
A retrusive contacts.
B. protrusive contacts.
C. mediostrusive side contacts.
D. laterotrusive side contacts.
E. lateral protrusive contacts.
138. Between which of the following permanent teeth is
the lingual embl'3sure smaller than the facial
embrasure?
A
B.
c.
D
Maxillary first premolar and maxillary second
premolar
Maxillary second molar and maxillal)! third
molar
Mandibular first molar and mandibular
second molar
Mandibular first premolar and mandibular
second premolar
139 On a mandibular first molar, the distofacial groove
serves as an escapeway for the mesiolingual cusp
of the maxillary first molar during which of the
following mandibular movements?
A. Working
B. Non-working
C. Protrusive
D. Centric slide
140. Which of the following primary grooves uniting in
the distal pit on the occlusal surface of the
mandibular second molar represents the one that
normally has no counterpan in the distal pit of the
first molar?
A. Distolingual
B. OF triangular
C. DL triangular
D. Distal marginal
E. Distal ponion of the central
141. Which of the following best describes the cervical
margin on the facial crown surface of the maxillary
first molar?
A. Straight
B. Evenly convex toward the apex
C. Evenly convex toward the occlusal
D. Irregularly convex toward the apex
E. Irregularly convex toward the occlusal
142. Which of the following represents the most
common type of root anomaly on the mandibular
first premolar?
A. Dwarfing
B. Elongation
C. Bifurcation
D. Concrescence
E. Trifurcation
143. Which of the following teeth represents the one
most likely to present with three roots?
A. Mandibular central incisor
B. Mandibular second premolar
C. Mandibular canine
D. Maxillary first premolar
E. Maxillary second premolar
16
144 Which of the following represents the structure in
the maxillary alveolar bone Hlat maxillary premolar
roots occasionally penetrate?
A. Antrum
B. Nasal septum
C. Frontal sinus
D. Zygomatic arch
E. Mandibular fossa
145. Which of the following jaw positions is determined
almost exclusively by tooth contact?
A. Rest position
B. Terminal hinge
C. Maximum opening
D. Maximum protrusive
E. Intercuspal position
146. In a CUSP TO FOSSA contacting relationship in
intercuspal position, the maxillary first premolar is
most likely to artiCUlate with which of the following
mandibular teeth?
A. Lateral incisor
B. Lateral incisor and canine
C. First premolar
D. Canine and first premolar
E. First premolar and second premolar
147. Which of the following is least likely to contribute to
or affect stability of the dental arch form?
A.
B.
C.
D.
E.
Periodontal health
Plane of occlusion
Occlusal contact forces
Interproximal contact form
Forces exened by the lips and tongue
148. When viewed from the frontal plane and
progressing posteriorly, the axial inclination of the
crowns of maxillary posterior teeth
A remains vertical.
B. inclines lingually.
C. inclines distally.
D. inclines mesially.
E. inclines buccally.
149. When viewed from the sagittal plane, the axial
inclination of the anterior teetll
A. remains venical.
B. inclines facially.
C. inclines mesially.
D. inclines distally.
E. inclines lingually.
15CJ The presence of m3melons on a 19-year-old
pCltient suggests which of the following conditions')
151.
A.
B.
C.
D.
E.
Fluorosis
!Vialformation
!Vialnutrition
Delayed el-uption
Anterior open bite
In osteomalacia, bones are weak because of the
failure of
A. remodeling of bone,
B, bone matrix formation,
C osteoblast proliferation
D, bone matrix calcification,
E, interstitial growth of cartilage,
152, Which of the following is the most common
location for an atherosclerotic induced aneurysm?
A. Thoracic aorta
B, Abdominal aorta
C. Coronary arteries
D_ Arch of the aorta
E, Common iliac arteries
153, Each of the following statements about poliovirus
infections is correct EXCEPT one, Which one is
the EXCEPTION?
A, Most infections are subclinical,
B_ Paralysis is an uncommon outcome of
infection,
C, There are 3 types of poliovirus, making 3
infections possible,
D, Virus is predominantly shed from the body
and transmitted in respiratory secretions,
E_ Some damaged neurons may be repaired,
restoring lost functions,
154 - Which of the following can result if an individual
having reactivation of latent varicella zoster virus
transmits virus to a seronegative individual?
A. Shingles
B. Chickenpox
C. Herpes labialis
D, Infectious mononucleosis
E, Herpetic gingivostomatitis
155. Which of the following conditions might be an initial
manifestation of early, acute HIV infection?
A. Kaposi's sarcoma
S, Wasting syndrome
C. Oral hairy leukoplakia
D_ Mononucleosis-like syndrome
E Pneumocystis car-inii pneumonia
17
156, Which of the following pathogens is the single
most common cause of sexuCllly transmitted
disease in the U_S,?
A.
B_
C
D,
E,
Treponema pallidum
Haemophilus ducreyi
Chlamydia trachomatis
Herpes simplex virus
Human immunodeficiency virus
157, Which of the following represents the mechanism
of action of diphtheria toxin?
A. Activates cAMP
B_ Causes cytolysis
C, Inhibits translation
D, Inhibits transcription
E, Inhibits DNA replication
158, Each of the following fluids is considered one that
can transmit HIV EXCEPT one, Which one is the
EXCEPTION?
159_
A. Semen
S_
Serum
C. Saliva
D_
Amniotic fluid
E, Breast milk
Which of the following represents the anaerobic
organism that is cultured from gingival scrapings
and that forms black colonies on hemin-containing
culture media?
A. Sarcina lutea
B_ Bacillus anthracis
C_ Veillonella alcalescens
D, Porphyromonas gingival is
E_ Alpha-hemolytic streptococcus
160, Beta 1-4 linkages connect N-acetyl glucosamine
and N-acetyl muramic acid, The resulting polymer
is found in bacterial
A flagella
B, capsules_
C, cell walls,
D, metachromatic granules_
161, Rough pneumococci that are grown in the
presence of DNA from smooth pneumococci
develop capsules, This process is ter-Illed
A_ translation,
B_ tranSduction,
C_ transformation,
D, conjugation,
1G2 During an outbreak of gastroenteritis caused by
Salmonella, a strain of the species suddenly
appears to be resistant to several antibiotics
Which of the following best explains why?
A. Parent strain has undergone several
mutations
B. Parent strain has acquired a plasmid
C There are several species of Salmonella
present, each resistant to one antibiotic
D. New strain is a species of Escherichia coli,
which has acquired virulence by mutation.
163. Which of the following karyotypes is found in
Turner syndrome?
A. 45, XO
B. 45, YO
C. 46,XX
D. 47,XXY
E.
47, XYY
164. Which of the following viruses are frequently
associated with eye infections?
A. Mumps and measles viruses
B. Parainfluenza and rubella viruses
C. Coxsackievirus and rhinoviruses
D. Adenoviruses and reoviruses
E. Herpes simplex virus and adenoviruses
165. In humans, the mumps virus is transferred by
A food.
B.
feces.
C. flies.
D. saliva.
166. Human leukocyte antigen (HLA) Class I molecules
are found on which of the following?
167.
A All nucleated cells
B. CD4+ T cells only
C. Epithelial cells only
D. HLA Class 11+ cells only
E. Mesencllymal cells only
Soft and hard tissue necrosis char'acterizes which
of the following fungal diseAses?
A. Mucormycosis
B. Cryptococcosis
C. Histoplasmosis
D. Coccidioidomycosis
E. Candidiasis
18
168. Nongonococcal urethritis is ofien caused by
microorganisms of whicll of the following genera?
A. Chlamydia
B. Treponema
C Neisseria
D. Hemophilus
169. Whicll of the following is least likely to produce
acute abdominal symptoms?
A. Cholelithiasis
B. Acute pancreatitis
C. Ampulla of Vater cancer
D. Carcinoma of the tail of the pancreas
170. Which of the following represents the arthritis that
is usually associated with aging?
A.
B.
C.
D.
Osteoarthritis
Gouty arthritis
Rheumatoid arthritis
Psoriatic arthritis
171. Multiple, lytic lesions of bone characterize each of
the following conditions EXCEPT one. Which is the
EXCEPTION?
A. Multiple myeloma
B. Metastatic carcinoma
C. Osteogenesis imperfecta
D. Hyperparathyroidism
E. Langerhans (eosinophilic) granulomatosis
172. Osteoporosis can be associated with each of the
following EXCEPT one. Which is the EXCEPTION?
A. Prolonged corticosteroid administration
B. Prolonged immobilization
C. Chronic malnutrition
D. Hypervitaminosis 0
E. Advanced age
173. In general, enzyme-deficiency diseases are
inherited by which of the following modes?
A
B.
C.
D.
Polygenic
X-linked dominant
Autosomal recessive
Autosomal dominant
174 Which of the following anemias results from dl'ug.
IIlduced bone-: malTOW suppression';
A.
Aplastic
B.
Sickle cell
C Pernicious
D Hemolytic
E.
Myelophthisic
175. Multiple drug resistance is associated with
A.
B.
C.
D.
E.
plasmids
recombination.
point mutations.
specialized transducing phages.
generalized transducing phages
176. Which of the following statements best describes
bacterial transformation?
A. DNA coding for RNA synthesis
8. RNA coding for protein synthesis
C. Acquisition of an inheritable trait by bacteria
mediated by DNA
D. Acquisition of an inheritable trait by bacteria
mediated by RNA
E. Binding of 30S and 70S ribosomes
177. Plasmid-mediated antibiotic resistance has been
observed in diseases caused by each of the
following EXCEPT one. Which one is the
EXCEPTION?
A.
B.
C
D.
E.
Streptococcus pyogenes
Staphylococcus aureus
Neisseria gono1717oeae
Borde/el/a pe/tussis
Haemophilus influenzae
178. Each of the following can produce a genotypic
change EXCEPT one. Which is the EXCEPTION?
179.
A.
8.
C.
D.
Conjugation
Lysogenization
Transformation
Exposure to tetracycline
TIle human immunodeficiency virus preferentially
infects which of the following cells?
A. NK
8. Helper I
C Cytotoxic T
D. Suppressor T
19
180. Which of the following viruses is most likely to be
isolClted from feces?
A. Rubella
B. Hepatitis C
C Influenza A
D. Coxsackievirus
E. Herpes simplex
181. Cellular tropism by viruses is dependent on whiCh
of the following?
182.
A.
B.
C.
D.
E.
Host cell DNA homology to viral DNA
Temperature of host cell incubation
Gamma-interferon production
Cell surface receptors
Viral enzyme synthesis
A 5-year-old child with vesicular lesions limited to
the palate and the posterior oropharyngeal mucosa
has an oral temperature of 101
0
F (380C). The
most probable diagnosis is
A. chickenpox.
B. herpangina.
C. recurrent oral herpes.
D. herpetic gingivostomatitis
E. hand-foot-and-mouth disease.
183. Which of the following cells are the most
radiosensitive?
A.
B.
C.
D.
E.
Neurons
Fibroblasts
Lymphocytes
Chondrocytes
Epithelial cells
184. During repair, which of the following substances is
essential for precollagen fibers to transfol-m into
collagen fibers?
A.
B.
C.
D.
E.
Cortisone
Carotene
Ascorbic acid
Prothrombin
Thromboplastin
185. Which of the following conditions represents an
intOXication rather than an infection?
A. Anthrax
B. Botulism
C. Chancroid
D. Bacteroidosis
E. Salillonellosis septicemia
1St! The tensile strength of a healing mucosal wound
depends on Wllich of the following')
A
B.
C.
D.
E.
Wound hormones
Epithelial regeneration
Formation of collagen fibers
Activation of fibrinolysis
Formation of elastic fibers
187. The chemotactic accumulation of leukocytes at the
site of immune complex deposition is a result of
A steroids.
B. histamine.
C. complement.
D. antihistamines.
188. Which of the following has the least ability to
regenerate?
A
B.
C.
D.
E.
Bone
Liver
Striated muscle
Collagen
Smooth muscle
189. Healing is retarded by each of the following
conditions EXCEPT one. Which one is the
EXCEPTION?
A
Trauma
B.
Immobilization
C.
Infection
D.
Hemorrhage
E.
Ischemia
190. Escape of plasma from capillaries into an area of
inflammation is favored by an increase in which of
the following?
A Hyaluronidase
B. Leukocyte migration
C. Vascular permeability
D. Osmotic pressure of plasma
E. Number of endothelial junctions
191. Each of the following conditions predisposes a
patient to develop cancer EXCEPT one. Whicll one
is the EXCEPTION?
A Asbestosis
B. Anthracosis
C. Hepatitis C
D. Gardner'S syndrome
E. Ulcerative colitis
20
192. Metastatic calcifications are most likely to be the
result of
A
B.
C.
D.
E.
gallstones.
atherosclerosis.
hyperparathyroidism.
osteogenic sarcoma.
lymphatic spread of lung cancer.
193 Difficulty in swallowing refers to
A dyspnea.
B. achalasia.
C dysphagia.
D. hemetemesis.
194. Each of the following viruses is potentially capable
of causing cell transformation EXCEPT one. Which
one is the EXCEPTION?
A.
B.
C.
D.
E.
Retrovirus
Herpesvirus
Picornavirus
Hepatitis B virus
Human papillomavirus
195. Which of the following terms refers to the presence
of digested blood in the stool?
A. Hemosiderosis
B. Hemochromatosis
C. Hematoma
D. Icterus
E. Melena
196. Chlamydia trachomatis might cause each of the
following diseases EXCEPT one. Which one is the
EXCEPTION?
A. Trachoma
B. Lymphogranuloma venereum
C. Inclusion conjunctivitis
D, Non-gonococcal urethritis
E. Primary atypical pneumonia
197. Which of the following conditions is most
commonly associated with acute pancreatitis?
A. Chronic alcohol abuse
B. Diabetes mellitus
C. Physical trauma
D. Viral infection
E. Hypercalcemia
198 Each of the following is a!!ributable to hepatic
failure EXCEPT one. Which one is the
EXCEPTION?
A Tremor
B Gynecomastia
C Mallory bodies
o Hypoalbuminemia
E. Spider telangectasia
199. Red hepatization refers to which of the following?
A. Early stage of acute hepatitis
B. Late stage of acute hepatitis
C. Cirrhosis caused by hemochromatosis
o Congestion of the liver caused by chronic
right sided heart failure
E. Stage of lobar pneumonia
200. Acute passive congestion often accompanies
which of the following'!
A. Edema
B. Inflammation
C. Contusion
O. Hematoma
21
NATIONAL BOARD PART 1 RELEASED ITEM KEY
ITEMS 101-200
101 B 126 0 151 0
102 B 127 A 152 B
103 C 128 C 153 0
104 0 129 0 154 B
105 A 130 B 155 0
106 0 131 B 156 C
107 A 132 B 157 C
108 A 133 B 158 C
109 C 134 C 159 0
110 B 135 E 160 C
111 C 136 C 161 C
112 E 137 A 162 B
113 E 138 0 163 A
114 E 139 B 164 E
115 E 140 B 165 0
116 C 141 0 166 A
117 0 142 C 167 A
118 0 143 0 168 A
119 C 144 A 169 0
120 C 145 E 170 A
121 0 146 C 171 C
122 0 147 B 172 0
123 B 148 E 173 C
124 A 149 B 174 A
125 A 150 E 175 A
23
176 C
177 A
178 0
179 B
180 0
181 0
182 B
183 C
184 C
185 B
186 C
187 C
188 C
189 B
190 C
191 B
192 C
193 C
194 C
195 E
196 E
197 A
198 C
199 E
200 A
NATIONAL BOARD PART 1 RELEASED ITEM KEY
ITEMS 1-100
1 B 26 0 51 C
2 B 27 B 52 B
3 0 28 B 53 E
4 C 29 0 54 A
5 A 30 E 55 0
6 C 31 0 56 B
7 B 32 B 57 A
8 8 33 8 58 8
9 E 34 8 59 8
10 C 35 0 60 0
11 A 36 A 61 0
12 8 37 C 62 A
13 E 38 C 63 C
14 8 39 E 64 A
15 A 40 A 65 8
16 8 41 A 66 E
17 E 42 A 67 8
18 C 43 E 68 E
19 E 44 0 69 A
20 0 45 A 70 0
21 C 46 0 71 D
22 0 47 E 72 B
23 8 48 C 73 A
24 A 49 0 74 8
25 8 50 C 75 B
76 C
77 0
78 0
79 B
80 0
81 0
82 E
83 C
84 0
85 0
86 C
87 A
88 C
89 E
90 C
91 C
92 C
93 B
94 C
95 0
96 A
97 B
98 E
99 A
100 0
The above key may be used to determine correct and incorrect answers; however, for this
item set, it is not possible to convert a raw score number correct to a standard score or
associated pass/fail status.
22

S-ar putea să vă placă și